Series 7

Lakukan tugas rumah & ujian kamu dengan baik sekarang menggunakan Quizwiz!

An investor who believes the U.S. dollar will strengthen against the Canadian dollar should profit from which of the following strategies? Buying puts on the Canadian dollar Writing puts on the Canadian dollar Writing a straddle on the Canadian dollar Establishing a call credit spread on the Canadian dollar

1 4 The investor who is bearish on the Canadian dollar should buy puts, write calls, and call spreads. Short straddles pay off when the market does not move either way.

Before effecting an initial penny stock transaction for a new customer, the registered representative must confirm whether the person is an established customer. obtain a signed risk disclosure document from the customer. obtain a signed suitability statement from the customer. determine suitability based on financial condition, investment experience, and investment objectives

2 3 4 According to the penny stock rules, registered representatives must provide risk disclosure information to all penny stock buyers, which customers must sign. In addition, they must determine suitability based on financial information, investor experience, and objectives supplied by the buyer. If an investor is not considered an established customer, they must sign a suitability statement, as well. In this case, we are told this is the initial trade by a new customer, so we are not going to confirm status as an established customer.

Cash dividends from real estate investment trusts (REITs) are A) taxed as ordinary income. B) taxed at a maximum rate for qualified dividends. C) not taxed. D) taxed as long-term capital gains.

A Cash dividends from REITs are taxed as ordinary income. A maximum rate for qualified dividends, which applies to qualified common stock dividends, does not apply to dividends from REITs.

A registered representative reproduced a research report prepared by an independent research analyst on his broker-dealer's letterhead, with no mention of the party who prepared the report. If this literature is forwarded to a select group of clients only, the registered representative's action is A) not allowed. B) allowed. C) allowed with the written approval of a principal of the broker-dealer. D) allowed only if the research report has been filed with FINRA.

A A broker-dealer is prohibited from presenting to a client research reports, analysis, or recommendations prepared by other persons or firms without disclosing that they were prepared by a third party.

A retiree is paid an annual amount equal to 30% of the average of his last five years' salary. Which of the following retirement plans offers this type of payment? A) Defined benefit B) Defined contribution C) Deferred compensation D) Profit-sharing

A A defined benefit retirement plan establishes, in advance, the payout to be received by the retiree. The formula is based on earnings and years of service.

Corporate bonds that are guaranteed are A) guaranteed as to payment of principal and interest by another corporation. B) guaranteed as to payment of principal and interest by the U.S. government. C) insured by Assured Guaranty Corporation. D) required to maintain a self-liquidating sinking or surplus fund.

A A guaranteed corporate bond is one guaranteed by another corporation that typically has a higher credit rating than the issuing corporation and is in a control relationship with it.

Which of the following statements regarding a bond trading flat is not true? A) It may be traded with accrued interest. B) It may be an income bond. C) It may have interest in arrears. D) It may be a bond in default.

A A municipal or corporate bond trading flat is trading without accrued interest. The bond may be an income bond, which normally pays no interest, or it may be a bond currently paying no interest because it is in default.

An investor purchased a corporate bond with a 6% coupon at a net price of 101. The bond had accrued interest for 45 days. Which of the following statements regarding the confirmation of this trade is correct? A) The total amount due on the purchaser's confirmation will appear as $1,017.50. B) The total amount due on the purchaser's confirmation will appear as $1,025. C) The total amount due on the purchaser's confirmation will appear as $1,010. D) The total amount received on the seller's confirmation will appear as $1,002.50.

A Accrued interest is always added to the price of a bond. When you buy the bond, you pay that accrued interest, and when you sell a bond, you receive that accrued interest. The principal value is 101, or $1,010. Forty-five days of accrued interest is ⅛ of a 360-day year, or ¼ of a 180-day semiannual interest payment. With a 6% coupon, the bond pays $30 every 6 months. One quarter of that is $7.50 so the total cost to the purchaser is the $1,010 plus the $7.50, or $1,017.50.

An investor with a relatively low risk tolerance for loss of principal wishes to make a long-term investment that will meet his needs and provide some protection against inflation. Which of the following mutual funds is likely the most suitable for him? A) A balanced fund B) A municipal bond fund C) An S&P 500 index fund D) A U.S. government bond fund

A Although suitability questions are rarely "cut and dried," we attempt to show you what the regulators are looking for. Balanced funds, with their mix of equities and fixed-income securities, offer some downside protection with the bonds and upside (inflation-beating) growth from the equities. Neither of the bond funds will protect against inflation. Furthermore, municipal bonds are never going to be a suitable choice unless something in the question refers to a high-tax-bracket investor. The index fund will surely provide the inflation protection, but as history demonstrated in 2008 and again in the spring of 2020, losses of 30% or more are possible.

A municipal revenue bond has a catastrophe call feature, but otherwise, is not callable. Which of the following statements regarding the features of this bond that must be described on a customer's confirmation is true? A) It need not be designated as callable. B) It must be designated as callable. C) It must be designated as callable in the event of an act of God. D) It must be designated as subject to eminent domain.

A Catastrophe call provisions associated with municipal revenue bond issues are not included on customer confirmations. Only call provisions with specific dates are included on confirmations.

All of the following are objectives in a direct participation program (DPP) except A) short-term capital gains. B) long-term capital gains. C) deductions against passive income. D) deferment of taxes.

A DPPs are used to defer present income into the future and take advantage of time. In doing so, any gains will be taxed at favorable long-term rates. The expected losses in the early years may be taken as deductions against passive income from other sources.

The term high-yield bond would apply to a bond with a Moody's rating of A) Ba. B) BBB. C) BB. D) Baa.

A High-yield bonds are those whose ratings fall below investment grade. Investment grade is the top four. Using Moody's descriptions, ratings run from Aaa to Aa to A to Baa to Ba to B and then below. The first rating below the top four is Ba. That is equivalent to a BB rating from Standard & Poor's (but the question asks specifically about Moody's).

All of the following statements regarding the tax treatment interest received from a collateralized mortgage obligation (CMO) investment is true except A) it is nontaxable at the local level. B) it is taxable at the federal level. C) it is fully taxable at all levels. D) it is taxable at the state level.

A Interest received from CMOs is fully taxable at the federal, state, and local levels. It is treated exactly the same as interest on a corporate bond. The only testable debt securities where there is a tax benefit are those issued by the U.S. Treasury (exempt from state and local [city] income tax) and those issued by municipal issuers where the interest is exempt from tax at the federal level and possibly exempt on the state and local level (in-state issuer), as well.

All of the following statements regarding liquidity are correct except A) it is the inability to find willing buyers for an asset. B) liquid assets include CDs and Treasury bills. C) the most liquid of assets is cash. D) a liquid asset can easily be converted to cash.

A Liquidity and marketability are often used synonymously. Liquidity is the ability to turn an asset into cash, whereas marketability is the ability to easily find buyers for an asset. If an asset is easily marketable, this would imply that it is also liquid.

As a registered representative, you have a customer that is concerned about losing principal when investing in a mutual fund. Because the customer is risk averse, you recommend a principal-protected fund. Which of the following statements is not correct? A) The portfolio is limited to bonds. B) The principal is guaranteed, less any front-end load charges. C) There is a lockup period before shares may be redeemed. D) The portfolio may contain both stocks and bonds.

A Most principal-protected funds investments are in both stocks and bonds, not just bonds. The principal is guaranteed, typically by an insurance policy. In order for the investor to receive the guarantee, the money invested is subject to a lock-up period, usually 5-10 years. If any redemptions take place during the lockup period, the investor would lose the guarantee. That could result in a loss.

Which of the following statements regarding negotiable certificates of deposit (CDs) are true? The issuing bank guarantees them. They are callable. Minimum denominations are $1,000. They can be traded in the secondary market. A) I and IV B) II and III C) II and IV D) I and III

A Negotiable CDs are issued primarily by banks and backed by the issuing bank. The minimum denomination is $100,000. These are sometimes referred to as jumbo CDs.

If interest rates fall, which of the following statements regarding collateralized mortgage obligations are true? Prepayment risk will increase. Prepayment risk will decrease. Prices of each tranche will rise. Prices of each tranche will fall. A) I and III B) I and IV C) II and IV D) II and III

A Prepayment risk is the risk that the underlying mortgages will be paid off sooner than expected. If rates fall, mortgage holders will refinance, paying off the existing high-rate mortgages with lower rate mortgages. Thus, a tranche with an expected average life of 5.5 years may be extinguished in two years because of an acceleration in prepayments. As rates fall, prices of the remaining tranches will rise.

Regular way settlement must occur on the second business day after the trade date for all of the following transactions except A) a broker-dealer buying a Treasury bond for its own account. B) a broker-dealer buying a corporate bond from another dealer. C) a customer selling a municipal bond through a broker-dealer. D) a customer buying closed-end fund shares through a broker-dealer.

A Regular way settlement for U.S. government bonds occurs on the business day after the trade date (T+1). Corporate bonds and closed-end funds settle regular way (T+2). Municipal Securities Rulemaking Board rules also require two-day settlement (T+2) of municipal bond secondary transactions.

Rule 144A regulates A) the sale of restricted stock to institutional investors. B) personal trading by research analysts. C) the sale of restricted stock by control persons. D) companies traded on the Nasdaq Global Select Market.

A Rule 144A regulates the trading of restricted securities to institutional investors known as qualified institutional buyers.

All of the following would affect a customer's equity balance except A) stock dividends. B) withdrawal of special memorandum account (SMA). C) cash dividends. D) interest charged.

A Stock dividends do not affect total equity in an account, only the number of shares contained (but at a lower per-share price). Cash dividends increase equity, while withdrawal of SMA and interest charges assessed against the account decrease equity.

Which of the following statements regarding the visible supply in The Bond Buyer is true? A) It is a daily listing of bonds to be offered in the next 30 days. B) It is the total of the bonds offered in the Blue List. C) It is a weekly listing of bonds sold in the past 30 days. D) It is a daily listing of available bonds.

A The visible supply implies that the supply of bonds will be available for the visible future.

Which of the following statements is true regarding IRA rollovers? A) One rollover may be conducted every 12 months, but it must be completed within 60 days. B) One rollover may be conducted every 12 months, but it must be completed within 90 days. C) One rollover may be conducted annually, but it must be completed within 60 days. D) One rollover may be conducted annually, but it must be completed within 90 days.

A The IRS permits IRA holders to engage in one rollover every 12 months. Legally, that is not the same as annually. To avoid taxation (and possible penalties as an early distribution), the rollover must be completed within 60 days. In a rollover from one IRA to another IRA, or from an employer-sponsored retirement plan to an IRA, the owner of the plan receives the funds. More popular is the direct transfer that occurs from institution to institution. Unlike the rollover, with a direct transfer, the one per 12-month limit is not in effect. Because the funds are never handled by the individual, another benefit is that the 60-day time limit does not apply.

One of your customers has made periodic purchases of shares of the Castel Growth Fund over the past several years. The customer has decided to take a profit and sell some of those shares. When the investor's tax return is prepared for the year in which the sale of those shares occurs, it is necessary to establish a cost basis of the shares sold. Which of the following methods is available for mutual funds, that is not available for determining the cost basis of stock? A) Average cost basis B) Dollar cost averaging C) Share identification D) FIFO

A The Internal Revenue Service allows using the average cost basis to determine the cost basis of redeemed mutual fund shares. Investors cannot use this method when selling shares of any security other than a mutual fund. The other methods of determining cost basis are FIFO and share identification. FIFO is the default method used by the IRS if an investor fails to choose. Share identification can frequently result in a lower tax bill, especially if the security was purchased at different intervals at varying prices.

If an investor has an established margin account with a current market value of $4,400 and a debit balance of $1,750 with Regulation T at 50%, how much buying power does the investor have in the account? A) $900 B) $4,400 C) $2,200 D) $2,650

A The Regulation T requirement is 50% of the current market value of $4,400, which equals $2,200. Equity equals the current market value of $4,400 minus the debit balance of $1,750, which equals $2,650. Excess equity is calculated by subtracting the Regulation T requirement of $2,200 from the current equity of $2,650, which equals $450. Buying power is then calculated by multiplying the excess equity of $450 by 2, which equals $900.

The Trust Indenture Act of 1939 covers all of the following securities transactions except A) a sale of an issue of $5 billion worth of Treasury bonds maturing in 2025. B) a public issue of debentures worth $60 million sold by a single member firm throughout the United States. C) a corporate bond issue worth $55 million sold interstate. D) a sale of an equipment trust bond issue worth $62 million.

A The Trust Indenture Act of 1939 requires all corporate debt issues of $50 million or more sold interstate to have a trust indenture. U.S. governments are exempt.

A corporation has an outstanding issue of 8% convertible debentures with a conversion price of $25. The bond indenture contains an antidilutive clause guaranteeing the debt holders the right to maintain proportionate equity conversion in the corporation. If the company pays a 10% stock dividend to its common shareholders, how will that affect the debenture holders? A) The bonds will now be convertible at approximately 22.73. B) The interest rate on the debentures will increase to 8.8%. C) Each debenture holder will receive a check for $100. D) They will receive four shares of the common stock.

A The antidilutive provision means the debenture holders will be able to convert into an equivalent share value as before. With a conversion price of $25, the bond is convertible into 40 shares ($1,000 ÷ $25). After the 10% stock dividend, they should be able to have 10% more shares, or 44 shares. That means the conversion price must be reduced. Divide $1,000 by 44 shares and the result is $22.73. Remember, anytime there is a stock dividend, prices go down.

If an investor in the 27% federal marginal income tax bracket invests in municipal general obligation public purpose bonds nominally yielding 4.5%, what is the tax-equivalent yield? A) 6.16% B) 5.72% C) 3.29% D) 16.67%

A The formula for computing tax-equivalent yield is: nominal yield ÷ (1 − federal marginal income tax rate). Let's plug in the numbers: 0.045 ÷ (1 − 0.27) = 0.045 ÷ 0.73 = 6.16%. This tells the investor that they would have to receive 6.16% interest on a taxable bond to have the same after-tax return as earning 4.5% tax-free. You can check it out by working backwards. If an investor receives 6.16% taxable, they will have to pay 27% (in this person's bracket) in income tax. That is a tax of 1.66% (6.16 x 27% = 1.66). Subtract 1.66 in tax from 6.16 and the result is 4.50%.

If a customer buys a Mount Vernon Port Authority municipal bond in the secondary market at 109 and holds the bond to maturity, what are the tax consequences? A) No capital gain or loss B) Capital loss of $9 C) Capital loss of $90 D) Capital gain of $9

A The investor's cost basis of bonds purchased at a premium is adjusted by amortization of the premium. In this case, there is a $90 premium that will have been completely amortized at maturity. At maturity, the adjusted cost basis equals the face value, and no loss or gain is realized.

Acme Pharmaceuticals previously had issued $200 million of common stock in an IPO. A year later, it issued $50 million of debentures at par value. Acme's leverage is what percentage of its total capital? A) 20% B) 400% C) 25% D) 50%

A The leverage is the extent to which borrowed funds make up the company's total capital. Total capital is the value of the equity and debt financing combined. Acme has issued $50 million of debentures (debt capital) and $200 million of equity capital (the common stock). That makes the total capitalization of Acme equal to $250 million. The leverage is $50 million divided by $250 million, or 20%. An analyst would consider this conservative leverage.

Typically, new municipal bond issues are sold to investors before the bonds are issued and available for delivery. An investor receives a when-issued confirmation describing the bonds. What information is included on this confirmation? A) The trade date B) The settlement date C) The accrued interest D) The total amount due

A The trade is known, so it is on the confirmation. Because the bonds are not available for delivery, the settlement date is unknown. Without a settlement date, accrued interest cannot be computed. Without accrued interest, there is no way to compute the total price.

Your firm has just assigned you a new client. Wanting to do the best job you can, you review the current investment holdings of the client. Included are the following mutual fund accounts: $50,000 in Class B shares of the STU Growth Fund $25,000 in Class A shares of the STU International fund $15,000 in Class A shares of the TUV Utility Fund STU and TUV offer rights of accumulation and breakpoints at $25,000, $50,000, and $100,000. If the client wishes to invest $20,000 into the Class A shares of the TUV Technology Fund, the sales charge would be based on A) the $25,000 breakpoint. B) the $50,000 breakpoint. C) the next computed net asset value. D) the $100,000 breakpoint.

A There are several important points in this question. Rights of accumulation provide that a new purchase is added to the value of existing accounts to determine the breakpoint reached. However, only Class A shares count because the Class B shares never paid a front-end load. Of course, only shares in the same fund family are used - we cannot combine STU shares with TUV shares. As far as the math, we have $15,000 in one TUV fund and are adding another $20,000. That brings the investor's account in TUV funds to $35,000, which is enough to meet the $25,000 breakpoint. Finally, the sales charge is computed as a percentage of the public offering price (POP), not the NAV.

An investor owns ten ABC 6s of 2045. The debentures have a conversion price of $50 with an anti-dilution provision. After ABC distributes a 20% stock dividend, the investor's position will be A) ten ABC 6s of 2045 with a conversion price of $41.67. B) ten ABC 6s of 2045 convertible into 16.67 shares. C) twelve ABC 6s of 2045 with conversion price of $50. D) ten ABC 6s of 2045 convertible into 20 shares plus forty additional shares.

A This question deals with the anti-dilution provisions of a convertible security. When there is a stock dividend or a stock split, the holder of the convertible maintains the same equity proportion as before. With a conversion price of $50, the debenture is convertible into 20 shares ($1,000 ÷ $50). After a 20% stock dividend, the holder should be able to acquire 20% more shares. That makes the security convertible into 24 shares. Divide the $1,000 par value by 24 shares and the conversion price is now $41.67.

Several years ago, one of your customers bought an original issue discount (OID) municipal bond at $960. The bond has now matured. For federal income tax purposes, the discount is A) tax free. B) taxed at maturity as ordinary income. C) taxed each year as ordinary income. D) taxed as a long-term capital gain.

A When buying an OID municipal bond, the discount must be accreted each year and treated as interest income. Because interest income from a municipal bond is tax free at the federal level, the discount is not taxed if the bond is held to maturity. If the customer had purchased at a discount in the secondary market, the discount would have been accreted and taxed as ordinary income.

Better Bond Sales (BBS) is the syndicate manager of new GO municipal bond issue. Should BBS sell some of the bonds, its earnings would be A) the spread. B) the total takedown. C) the spread plus the manager's fee. D) negotiated between the issuer and the manager.

A When the syndicate manager makes the sale of a new issue of bonds, the manager earns the entire spread (the difference between the amount paid to the issuer and the offering price). The manager's fee is included in that spread. The syndicate members earn the total takedown on bonds they sell.

Disregarding commissions, and investor purchasing $10,000 face amount of Treasury notes at a price of 98.12 would expect to pay A) $983.75. B) $9,837.50. C) $981.20. D) $9,812.00.

B Please note that the purchase is not for $1,000, but for $10,000. Treasury notes (and bonds) are quoted in 32nds. This quote of 98.12 is 98 12/32 or 98 3/8% of $10,000.

FINRA rules require broker-dealers to conduct anti-money laundering training A) biannually. B) on an ongoing basis. C) annually. D) quarterly.

B Rather than set a fixed schedule, FINRA rules require that anti-money laundering training be conducted on an ongoing basis.

All of the following statements regarding municipal bond official statements are true except A) a retail customer must receive an official statement no later than the settlement date. B) an official statement must be delivered only upon request of a retail customer. C) all retail purchasers of a new municipal bond issue must receive a final official statement. D) a municipal securities broker-dealer may satisfy the delivery requirements by providing a n

B A final official statement must be delivered to retail buyers of a new issue on or before the settlement date. With today's technology, most investors receive their official statement through EMMA.

A customer who has, as part of her account holdings, unlisted REITS, as well as a limited partnership interest in an oil and gas program, may expect her servicing member firm to show A) an exact per share value as calculated on the last business day of the month. B) a per share estimated value of the securities. C) the amount shown on Tape B the business day before the account statement closing date. D) no valuation for the unlisted REITS and the original investment made in the DPP interest.

B A general securities member must include in a customer account statement a per share estimated value of a DPP or unlisted REIT security, in a manner reasonably designed to ensure that the per share estimated value is reliable.

Given the current business climate, an investor believes that a number of industries will be going through a consolidation over the next two to three years. Willing to invest $30,000 in the opportunity to profit if the consolidation occurs, which of the following would be the most suitable recommendation? A) Corporate bond fund B) Special situation fund C) Buy call options on select stocks D) Sector fund

B A special situation fund can be specific to mergers and acquisitions within a particular industry or among many and would be a suitable choice, given the investor's opinion that consolidation may occur. Sector funds focus on only one industry or area, and corporate bond funds would have no advantages in cases of industry consolidation. Purchasing standard option contracts on select companies would be extremely speculative, and their nine-month life cycle would require that the positions be reinstated over a two- to three-year period, adding to commission costs.

The longest initial maturity for U.S. T-bills is A) 39 weeks. B) 52 weeks. C) 2 years. D) 13 weeks.

B As money market instruments, the longest initial maturity of Treasury bills is 52 weeks. Those bills are auctioned once a month. T-bills of shorter maturities are auctioned weekly. The shortest initial maturity is four weeks.

On September 1, an investor sold 100 shares of KLP Corporation common stock for a loss of $1 per share. On September 15, he purchased a KLP convertible bond with a conversion price of $40. How much of the original loss may he now declare for tax purposes? A) None B) $75 C) $40 D) $100

B Because he purchased the convertible bond less than 30 days after realizing the loss, the sale of the stock falls under the wash sale rule. Investors who sell a security at a loss, and repurchase it, including its equivalent (e.g., convertible bond, warrant, or call option), 30 days before or after the sale will have the loss disallowed by the IRS. With a conversion price of $40, the bond could be converted into 25 shares (1,000 / 40) of KLP common stock. Hence, the investor has "bought back" the equivalent of 25 shares and may only declare a $75 loss, as the remaining $25 loss will be disallowed. Look at this question as if it said, "On September 15, he purchased 25 shares of KLP stock." That washes out $25 of the loss, but the rest is okay.

A 38-year-old investor places $25,000 into a qualified single premium deferred variable annuity. Twenty years later, with the account valued at $72,000, the investor surrenders the policy. If the investor is in the 25% marginal income tax bracket, the total tax liability is A) $18,000. B) $25,200. C) $11,750. D) $16,450.

B Because this is a qualified annuity, the entire withdrawal is taxable. The surrender value of $72,000 has a cost basis of $0.00. That $72,000 is taxed at the marginal rate of 25%. Furthermore, because the investor is younger than 59½ (38 + 20 = 58), there is the additional 10% penalty tax. Effectively, this is a 35% tax on $72,000.

SEC regulations for securities issued by investment companies prohibit which of the following? Closed-end funds from issuing preferred stock Open-end funds from issuing preferred stock Closed-end funds from issuing bonds Open-end funds from issuing bonds A) II and III B) II and IV C) I and III D) I and IV

B Closed-end funds may issue more than one class of security, including debt issues and preferred stock. Open-end funds may issue only one class of security: redeemable, voting common stock. They may not issue senior securities.

A customer wants to be a day trader but is interested in the term pattern day trader and asks you to define the term. You state that all of the following are true of pattern day traders except A) they make four or more day trades in a five-business day period. B) the buying power in margin accounts is the same as for other customers. C) they must have a minimum of $25,000 of equity in their account on any day in which trading occurs. D) in a margin account the minimum maintenance requirement is 25%.

B Pattern day traders are also treated differently when it comes to buying power. Buying power for day traders is four times the maintenance margin excess. Maintenance margin excess is defined as the equity in the account above the 25% minimum requirement. For regular customers, buying power is double SMA.

All of the following would be considered either retail communications or correspondence except A) a letter to 10 individual investors within the past week regarding a new investment strategy. B) an email to several municipalities sent out in a single day offering your firm's services for underwriting their municipal securities. C) a written communication to all of the firm's customers regarding a new mutual fund being offered. D) an electronic communication distributed through the firm's website regarding potential opportunities with the firm as a registered representative.

B Communications with government entities, which includes municipalities, fall under the heading of institutional communications. The others are all examples of either retail communications or correspondence, depending on how many recipients there are within a 30-calendar-day period. (Retail equals more than 25 retail investors within any 30-calendar-day period, and correspondence is 25 or fewer retail investors within any 30-calendar-day period.)

All of the following accounts are permitted to write calls except A) a mutual fund against a long stock position. B) a corporation against its own stock. C) a custodian in an UTMA account against a long stock position. D) an individual in a margin account.

B Corporations are not permitted to write calls against their own stock. If exercised, they would have to issue shares at the strike price, and this would have a dilutive effect on shareholders.

Depletion allowances in oil and gas programs are based on the amount of oil A) lost to shrinkage. B) sold. C) extracted. D) in reserve.

B Depletion allowances are allowed to compensate for a mineral resource, which is considered accomplished when it is sold.

When conducting a discussion with a client about the merits of investing in a direct participation program, all of the following could be tax advantages except accelerated depreciation. depletion allowances. recapture of depreciation. tangible drilling expenses. A) II and III B) III and IV C) I and IV D) I and II

B Depreciation is the deduction against income representing the cost recovery of certain fixed assets. When one of those assets is sold for more than the straight-line depreciated value, the excess is recaptured as ordinary income. Only intangible drilling expenses benefit the limited partner.

SEC rules require that open-end management companies distribute dividends to their investors from the firm's A) portfolio earnings. B) net investment income. C) capital gains. D) gross revenue.

B Dividends must be paid from the net investment income.

The rules on opening an options account contain a number of differences from the normal cash account at a broker-dealer. One of those differences is, if applicable, A) the requirement to obtain the signature of the principal approving the account. B) the requirement to obtain the signature of the registered representative handling the account. C) obtaining the name of the customer's employer (if employed). D) the need to determine if the customer is of legal age.

B For a normal account, FINRA requires the signature of the principal approving the account, but not that of the registered representative who will be handling that account. For options, that additional signature is necessary. The other choices are required on any new account form, options or not.

Compared to defined contribution plans, defined benefit plans give the highest return to employees who are highly compensated. receive lower compensation. have fewer years until retirement. have many years left until retirement. A) II and IV B) I and III C) II and III D) I and IV

B Highly compensated employees who have fewer years until retirement will experience advantages over other employees with this type of plan. Their retirement benefits are predefined and generally linked to the compensation level they attained while employed. After a short time with the company, a person may qualify for benefits comparable to those it would have taken many years to attain under a defined contribution plan.

Which of the following order types are available to customers for use in NYSE equity markets? Fill or kill (FOK) Immediate or cancel (IOC) All or none (AON) Order cancels other (OCO) A) I and IV B) II and IV C) II and III D) I and III

B IOC and OCO orders are available to customers for use in the NYSE equity markets. FOK and AON orders are no longer permitted in NYSE equity markets.

A new offering has a green shoe option. This means A) the syndicate is obligated to purchase up to 15% of the offering. B) the syndicate can oversell by up to 15% of the offering. C) the syndicate members have purchased put options to protect against a decline in the price of the stock. D) the issuer has purchased put options to protect itself against a decline in the price of the stock.

B If the syndicate manager, based on anticipated demand, wants to sell more shares than initially registered with the SEC, the manager can invoke the green shoe clause on short selling. A green shoe clause, negotiated with and agreed to by the issuer, allows the syndicate to sell up to 15% more shares than initially registered within 30 days of the IPO beginning to trade. The additional shares are made available to the syndicate by the issuer. To be effective, a green shoe clause must be disclosed in both the registration statement filed with the SEC and the prospectus. **This question deals with material not covered in your LEM, but it relates to recent rule changes and/or student feedback.

Your customer has purchased 100 shares of Synovial Lubrication Products (SLP) at $95 per share. The date of the purchase was April 22, 2021. Simultaneously, the customer purchased one SLP Dec 90 put for 3. At the expiration date of the option, SLP's market price is $101 and the option expires unexercised. What is the customer's cost basis in SLP? A) $101 per share B) $98 per share C) $95 per share D) $92 per share

B If, on the same day, a customer buys stock and buys a put option on that stock as a hedge, the put is said to be married to the stock. For tax purposes, irrespective of what happens to the put, the cost basis of the stock is adjusted upward by the premium paid. Even if the put expires worthless, there is no capital loss on the put. Rather, the premium paid is reflected in the cost basis of the stock. Therefore the initial cost of $95 per share is now increased by the $3 premium paid resulting in a new cost basis of $98 per share.

A broker-dealer can rehypothecate (repledge) up to A) 140% of the equity in a customer's margin account. B) 140% of the debit balance in a customer's margin account. C) 50% of the equity balance in a customer's margin account. D) 50% of the debit balance in a customer's margin account.

B In a margin account, hypothecation is the pledging of customer securities as collateral for the margin loan the customer will receive. The broker-dealer repledges (rehypothecates) the securities as collateral to the lending bank. Broker-dealers are permitted to pledge up to 140% of the debit balance in the customer's account.

The Interstate Bridge Authority has an outstanding revenue bond. For the most recent operating period, the Authority has net revenue of $36 million, operations and maintenance expenses of $16 million, debt service requirements of $18 million, and surplus funds of $2 million. The debt service coverage ratio for the Interstate Bridge Authority's revenue bond is A) 1:1. B) 2:1. C) 1:11:1. D) 2.25:1.

B In the absence of a described revenue pledge (net or gross), the net revenue pledge should be used. This means that the debt is serviced after the expenses for operations and maintenance have been satisfied. The net revenue of the project is revenues after subtracting those expenses. In this question, that is the $36 million figure given. The debt service coverage ratio is determined by dividing the net revenue by the debt service requirement. In this question, the debt service coverage ratio would be 2:1 ($36 million divided by $18 million = 2). If you subtracted the $16 million of expenses because you did not notice that we gave you the net revenue, your ratio was 20 divided by 18 = 1.11 to 1. If you added the surplus (not an expense), your ratio was 18 divided by 18 = 1:1. It is not uncommon to have information in a question that is not needed to arrive at the solution.

A customer sells short 1,000 XYZ at 60. Three months later, XYZ is at 44. Which two of the following strategies are the most likely the customer would use to protect her unrealized gain? Sell 1,000 XYZ 45 stop Buy 1,000 XYZ 45 stop Buy 10 XYZ Mar 45 calls Buy 1,000 XYZ 45 stop limit A) I and IV B) II and III C) II and IV D) I and III

B In this short position, the customer currently has an unrealized gain of 16. She stands to see her unrealized gain begin to erode if the stock price rises, so she could enter a buy stop order above 44 to allow herself to buy and cover her position if a price rise occurs. Purchasing calls would also be effective, because the right to exercise would allow the investor to buy stock at 45 and protect a gain of 15 points less the premium paid. If the buy stop at 45 is correct, why isn't the buy at 45 stop limit correct as well? Good question. Remember, when a stop limit order is triggered, the order becomes a limit order. When the stock rises to 45 (or higher), the customer now has a buy at 45 limit which means pay no more than 45. Once triggered, the stock may never get as low as 45 and the customer's order to buy will not be executed.

Which of the following best describes an intangible drilling cost? A) Proven reserve of oil or gas B) Labor, fuel, or drilling rig rental C) Tax liability D) Exploratory well drilling

B Intangible drilling costs are the noncapital costs of putting in a well. They are currently deductible expenses such as fuel, wages, and rent. An intangible drilling cost is one that, after expenditure, has no salvage value.

Intangible drilling costs would include all of the following except A) wages. B) casing. C) fuel. D) land surveys.

B Intangible drilling costs are those associated with drilling a well, but do not include the cost of capital equipment (e.g., pumps, casing). They include wages, fuel, repairs, hauling, supplies, surveys, tests, and drilling mud, and they are incidental to and necessary for the drilling activity.

The alternative minimum tax (AMT) is designed to present an alternative tax computation that disallows deductions for certain tax preference items and includes certain nontaxable income. Which of the following is not a tax preference item? A) Tax-exempt interest received on private purpose bonds B) Interest received on corporate bonds C) Local income and property taxes D) Certain costs associated with an oil and gas drilling program

B Interest on corporate bonds is taxable and included in an investor's adjusted gross income (AGI), but not for the AMT. Tax-exempt interest on private activity bonds and excess intangible drilling costs in an oil and gas DPP are included as tax preferences. In addition, state and local taxes and accelerated depreciation are in the list of preference items.

Tamika is a registered representative with Financial Engineers, LLC, a FINRA member broker-dealer. The firm uses an investment policy statement (IPS) to help design financial plans for their clients. One of Tamika's current clients plans to purchase a new boat seven months from now. When using the IPS, this would be considered A) a long-term goal. B) an investment constraint. C) an investment objective. D) a financial objective.

B Investment constraints are obstacles or restrictions that must be met in order to meet goals. In this case, we are dealing with a liquidity constraint—in seven months, cash will be necessary to make the purchase.

A pension plan might invest in each of the following except A) variable annuities. B) tax-free municipal bonds. C) equities. D) corporate bonds.

B It is inappropriate to place tax-free investments into a tax-deferred plan because there is no benefit to the deferral.

Under the 5% markup policy, which of the following determines the amount of markup in a principal transaction? A) Lowest bid B) Lowest ask C) Highest ask D) Highest bid

B Markups are always based on the inside offer, which is the lowest ask price in a particular security. Markdowns are based on the inside bid, which is the highest bid price for a particular security.

If a company issues $10 million in par value convertible debentures, all of the following balance sheet items will be affected except A) working capital. B) net worth. C) assets. D) liabilities.

B Net worth is not affected by the issuance of long-term debt because it does not represent ownership. Assets will be affected (increased) by the issuance of long-term bonds. Liabilities will be affected (increased) by the amount of the issuance. Working capital will also increase. How does the working capital increase if the net worth does not? Because this company now has $10 million more in cash, and the liability is the debenture that is included in long-term debt, not current liabilities.

One of the features of variable insurance products is the ability to withdraw money from the policies. Which of the following statements is correct? A) Withdrawals from variable annuities are taxed on a FIFO basis, while those from variable life are taxed on a LIFO basis. B) Withdrawals from variable annuities are taxed on a LIFO basis, while those from variable life are taxed on a FIFO basis. C) Withdrawals from both are taxed on a FIFO basis. D) Withdrawals from both are taxed on a LIFO basis.

B One advantage to withdrawing cash value from a variable life insurance policy is that it receives FIFO treatment. That means there is no tax until the withdrawal reaches the cost basis (premiums paid) of the policy. With annuities, the taxation is LIFO. Therefore, the first money withdrawn is taxable. In addition, if the policyowner is not yet 59, the 10% penalty applies. **This question deals with material not covered in your LEM, but it relates to recent rule changes and/or student feedback.

A walk-in customer completes the new account form and includes all of the information required by the customer identification program. However, the customer supplies none of the requested financial data and is unwilling to discuss objectives. Under Regulation BI of the SEC, A) the account may be opened, but all recommendations must be suitable based on the customer's situation. B) the account may be opened, but no recommendations may be made. C) opening this account could place the firm in the position of violating Regulation BI. D) the account may be opened, but only after receiving SEC approval.

B Regulation BI deals with recommendations from a broker-dealers to its customers. As long as there are no recommendations, the rule does not apply. When the customer refuses to supply the necessary suitability information, the account may be opened, but trading must be limited to unsolicited orders. ** This question deals with material not covered in your LEM, but it relates to recent rule changes and/or student feedback.

Regulation T controls the extension of credit from A) broker-dealer to broker-dealer. B) broker-dealer to customer. C) bank to broker-dealer. D) bank to customer.

B Regulation T controls the extension of credit from broker-dealer to customer, with customer securities providing the collateral for such loans.

All of the following communications are exempt from filing with FINRA except A) retail communications previously filed with FINRA. B) retail communications that make a financial or investment recommendation. C) communications that refer to an investment solely as part of a listing of products offered by the member. D) prospectuses and preliminary prospectuses.

B Retail communications previously filed with FINRA, prospectuses and preliminary prospectuses, and communications that refer to an investment solely as part of a listing of products offered by the member are all exempt from filing with FINRA. Any retail communication that makes a financial or investment recommendation would require filing.

Which two of the following statements regarding a customer's account profile are true? New account information must be sent to the customer for verification within 15 days of opening. New account information must be sent to the customer for verification within 30 days of opening. The customer's profile must be sent for updating and verification no less frequently than every 24 months. The customer's profile must be sent for updating and verification no less frequently than every 36 months. A) I and IV B) II and IV C) II and III D) I and III

B SEC Rule 17a-3 requires delivery of a copy of the account information within 30 days of opening (and every 36 months thereafter). Customers are to verify the information and note any relevant changes to the information.

One of your customers invests $20,000 in an oil and gas limited partnership program. The Schedule K-1 she receives at year-end from the partnership indicates that operating revenues and operating expenses were exactly the same. In addition, her share of the year's depletion allowance is $6,000. During the year, she received a cash distribution of $8,000. What is her basis as of year-end? A) $16,000 B) $6,000 C) $8,000 D) $12,000

B She began with a basis of $20,000 (her original investment). During the year, she received a distribution of $8,000. That lowered her basis (the amount of money "at risk") to $12,000. In addition, the depletion represents a nonoperating expense that can be taken as a loss. That brings the basis down to $6,000.

In a new margin account, if a customer buys 300 XYZ at 48 and simultaneously writes 3 XYZ Jan 50 calls at 1, the Regulation T margin requirement is A) $6,900. B) $7,200. C) $7,500. D) $7,350.

B The Regulation T requirement for purchasing $14,400 (300 × 48) of stock is 50%, or $7,200. The Regulation T requirement for writing covered calls is zero. Therefore, the Regulation T requirement for establishing both of these positions is $7,200. Note that this question asks for the Regulation T requirement, not the deposit that must be made. The margin call (deposit) would be $6,900 because the requirement is reduced by the $300 premiums already received into the account for the calls. After depositing $6,900, the customer will have $7,200 in the account, which meets the requirement.

Under the USA PATROIT Act of 2001, which of the following must be maintained by financial institutions, such as banks and broker-dealers, to prevent the financing of terrorist operations and money laundering? A) Do-not-call lists B) Customer identification programs (CIPs) C) Specially Designated Nationals and Blocked Persons list D) Privacy notices

B The USA PATRIOT Act of 2001 requires financial institutions to maintain CIPs to protect against financing terrorist operations or activities and potential money laundering activities. The Office of Foreign Asset Control (OFAC) publishes and maintains the Specially Designated Nationals and Blocked Persons list, which financial institutions use to determine if any customers or potential customers have been identified by OFAC as posing a terroristic threat or are involved in money-laundering activities.

All of the following statements about targeted amortization class (TACs) CMOs are correct except A) in exchange for higher price risk, they generally offer a slightly higher interest rate. B) they are the most volatile of the tranches. C) they have high extension risk. D) they have transferred prepayment risk to companion tranches.

B The Z-tranche, just like a zero-coupon bond, is the most volatile. TACs transfer the prepayment risk to companion tranches like PACs do, but they retain extension risk. This causes greater price risk in the market that is compensated for with a somewhat higher interest rate.

An investor purchases an original issue discount general obligation municipal bond (OID) on the offering and holds it to maturity. The IRS treats the accretion of the discount as A) long-term capital gain. B) tax-free interest income. C) taxable interest income. D) short-term capital gain.

B The discount on an OID municipal bond is considered the bond's interest. Because interest on a GO municipal bond is tax free, when that interest is ultimately paid at maturity date, the tax treatment is the same as if interest was paid semiannually. There is a more complicated situation when the OID bond is purchased in the secondary market or when regular municipal bonds are purchased at discount in the market.

If an officer of a bank wants to purchase new issues, which of the following statements is true? A) She may purchase a new issue because no banking rules prohibit it. B) She may not purchase a new issue because she is considered a restricted person. C) She may purchase a new issue because anyone is allowed to purchase new issues. D) She may not purchase a new issue unless the amount she wishes to purchase is considered small in relation to the total offering.

B Under the rules regarding the purchase of new issues, bank officers would be characterized as restricted persons. They may not, therefore, purchase new issues.

An investor has the following tax picture in 2021: Tax loss carryover from 2020: $9,000 Capital gains realized in 2021: $15,000 Capital losses realized in 2021: $2,000 What is the investor's reportable gain or loss for 2021? A) $6,000 net capital gains B) $4,000 net capital gains C) $13,000 net capital gains D) $13,000 net capital gains and a $4,000 loss carryover into 2022

B The first thing to remember is that there is no limitation on the amount of capital loss that may be carried over to use against capital gain. The $3,000 limitation is against income. In determining an investor's capital gain or loss for the tax year, all gains and losses must be aggregated and offset against each other. In this situation, all of the prior year's loss carryover of $9,000 is added to the current year's loss of $2,000. The total loss of $11,000 is offset against the total capital gains of $15,000, for a net capital gain of $4,000.

The DERP Corporation has a rights offering. The common stock is currently selling at $45.50. DERP is issuing one new share of stock at $40 per share for each 10 shares owned. What is the theoretical value of one right when the stock is traded ex-rights? A) $0.40 B) $0.55 C) $0.50 D) $0.45

B The formula for the theoretical value of a right when it is ex-rights (the buyer doesn't get the rights) is (M ‒ S) ÷ N where M = market price of the stock, S = the subscription price, and N = number of rights needed. Plug in the numbers and you have ($45.50 ‒ $40) divided by 10. That is $5.50 divided by 10 or $0.55 each

Your broker-dealer acts as a prime broker for ABC Fund. In this arrangement, your broker-dealer is likely providing which of the following services? Execution of all transactions for the fund portfolio Clearing services Lending for trades done on margin Ensuring that all exchange trading rules are complied with A) I and III B) II and III C) II and IV D) I and IV

B The prime broker would supply clearing services and lending services for a marginable transaction, as well as back-office support such as cash management, account statements, and transaction processing. Actual executions and abiding by all exchange rules when transactions occur is the responsibility of the executing broker-dealers.

When a customer instructs a registered representative to transfer and ship, the representative instructs the margin department to transfer ownership into A) the brokerage firm's name and deliver the securities to the customer. B) the customer's name and deliver the securities to the customer. C) the customer's name and deliver the securities to the customer's bank for safekeeping. D) the brokerage firm's name and deliver the securities to the brokerage firm's commercial bank for safekeeping.

B The term transfer and ship means to transfer the securities into the name of the customer and ship (deliver) the securities to the customer. To hold in street name would require the securities to be transferred into the name of the broker-dealer and held for safekeeping.

An investor purchased 100 shares of a stock three years ago at $38 per share. Disappointed with the stock's performance, the investor sells it for $35 per share. Two weeks later, after the company announced higher-than-expected earnings, the investor purchased 100 shares at $44 per share. When this investor decides to sell the newly purchased shares, the cost basis will be A) $38 per share. B) $47 per share. C) $41 per share. D) $44 per share.

B This is a wash sale situation. Selling a stock at a loss and repurchasing it within 30 days "washes" out the loss for current tax purposes. The loss, in this case $3 per share, is added to the cost of the repurchased stock. Thus, $44 plus $3 equals a new cost basis of $47 per share.

A registered representative is reading an article in a popular magazine about the advantages of tax deferral in retirement planning. There is a note that reprints of the article are available. In order to send these reprints to existing and prospective customers, A) approval by a principal is required within 10 days after first use. B) member alterations to the contents are only to make it consistent with applicable regulatory standards or to correct factual errors. C) the name of the underwriter who commissioned the article must be prominently displayed. D) filing a copy with FINRA is required.

B This is an example of an independently prepared reprint. It is a form of retail communications and can be used only if the preparer is independent of the member firm. In most cases, these are used "untouched." However, if there are factual errors or statements contrary to FINRA standards, they must be fixed. Preapproval by a principal is required and there is no filing necessary with FINRA. If the publisher is independent but received money from an issuer or underwriter for authoring the article, it may not be used.

Which of the following is an automated system of delivering information relating to the market for municipal securities? A) The Bond Buyer B) Thomson's Muni News or Muni Market Monitor C) INSTINET D) The Blue List

B Thomson's Muni News or Muni Market Monitor (formerly Munifacts) supplies up-to-the-minute information to its subscribers.

Which of the following is required to sign a new account form for a cash account? A) The spouse of the customer B) The principal C) The customer D) The registered representative

B To open a cash account, only the signature of the principal accepting the account is required. For margin accounts, the signature of the customer is required on the margin agreement. The signature of the spouse is required only for a joint account.

One of your customers is interested in purchasing the shares of a new issue from a local manufacturing company. The issue is for $15 million of common stock. The investor's net worth is $95,000 and net income is $75,000 per year. The plan is to invest $15,000 into this stock. Under Regulation A of the Securities Act of 1933, this investment is A) not permitted because Regulation A requires investors to be accredited. B) permitted. C) not permitted because it represents more than 10% of the customer's net worth. D) not permitted because it represents more than 10% of the customer's net income.

B Under Regulation A, an offering of $20 million or less in a 12-month period is a Tier 1 offering. Unlike a Tier 2 (up to $75 million) offering, there are no restrictions based on net worth or net income. A separate question, but not relevant to this one, is the suitability of this investment. It is important to stick with the question being asked.

During the cooling-off period, a registered representative (to highlight key points) marks a preliminary prospectus and sends it to a client. This action is A) permitted without restriction. B) prohibited. C) permitted if approved by a principal. D) permitted if the customer is an accredited investor.

B Under no circumstances may a registered representative mark a preliminary or final prospectus.

Which of the following investments is the most liquid? A) Common stock traded on the New York Stock Exchange B) Money market mutual funds C) Variable annuities D) Foreign stock mutual funds

B When an asset is liquid, the owner can quickly turn it into cash. If you need money to pay bills today or you are going shopping, you need a high degree of liquidity. Let's look at the answer choices. Mutual funds of any type must comply with federal law requiring them to honor a redemption request at the next computed net asset value per share. Then, those proceeds must be sent within seven days. If the bill collector is at your door or the sale ends today, that's not quickly enough. The same federal law applies to the variable annuity. Listed stock has great liquidity, but in most cases, settlement is T+2 meaning you'll have to wait a couple of days to get the money. Even if the sale was made in what is called "delivery for cash" (same day settlement), you'll have to wait for the check from the broker-dealer or make the trek to the office. Even then, the funds won't generally be available to the investor until the next business day. On the other hand, when it comes to the money market mutual fund, invariably, you have a checkbook, no different from the checkbook at your bank. That means, if you have to pay the bill collector today or take advantage of the last day of a sale, you simply write a check. There is no investment that gets you your money faster than that.

Yield quotes on collateralized mortgage obligations (CMOs) are based on A) the underlying mortgage's maturity. B) the tranche's expected life. C) the underlying mortgage's average life. D) the underlying mortgage's interest rate.

B Yield quotes on CMOs are based on the tranche's expected life, not the average life of the mortgages in the pool backing all of the tranches.

If a high-income customer is subject to alternative minimum tax (AMT), which of the following preference items must be added to adjusted gross income to calculate his tax liability? A) Income from a municipal security issued to finance parking garages B) Distributions from a corporate bond mutual fund C) Interest on a private-purpose municipal bond D) Interest on a municipal bond issued to finance highway construction

C If more than 10% of a bond's proceeds go to private entities, the interest on the bond is a tax preference item for AMT purposes.

An example of a taxable bond issued by a municipal government is A) a Build America Bond (BAB). B) a tax anticipation note (TAN). C) Series EE bonds. D) a general obligation bond (GO).

BABs are municipal issues created under the Economic Recovery and Reinvestment Act of 2009 to assist in reducing the cost of issuing municipalities and to stimulate the economy. Bonds to fund municipal projects have traditionally been sold in the tax-exempt arena, but BABs are taxable obligations.

An investor purchased a single premium deferred variable annuity 20 years ago. The premium deposit was $50,000. The account is now worth $200,000 and the investor is still working. When does the investor have to begin taking required minimum distributions? A) At age 72 or when no longer working, whichever is later B) At age 59½ C) Never with a nonqualified annuity D) At age 72

C On the exam, unless stated to the contrary, every annuity is nonqualified. One of the benefits of nonqualified annuities is that there is no age at withdrawals must commence. In general, earnings withdrawn prior to age 59½ are subject to the additional 10% penalty on top of tax at ordinary rates.

Which of the following investment companies is limited to offering investors a single class of common stock representing ownership in the company? A) A unit investment trust B) A closed-end management company C) A mutual fund D) A face amount certificate company

C A mutual fund is an open-end management company that raises capital solely through the issuance of a single class of common stock. A face amount certificate sells interests in a pool of bonds that all mature on the same date. A unit investment trust is a fixed portfolio of debt or equity securities that sells redeemable units, not shares, to investors. Those units represent the investor's share in the trust's assets. A closed-end management company does a one-time IPO of common stock, after which its shares trade in the secondary market just like any corporate stock. Closed-end companies can also have a preferred stock issue and a bond issue. Please note: The term "single class" of stock is not related to the different classes for sales charge and expense purposes. Those classes merely define the costs the investor will incur to acquire and own the shares.

An investor purchased an interest in a limited partnership, paying $10,000 in cash and signing a recourse note to the partnership under a letter of credit for $40,000. Which of the following statements are true? The investor's tax basis will be $10,000. The investor's tax basis will be $50,000. The investor's maximum loss will be $10,000. The investor's maximum loss will be $50,000. A) I and IV B) I and III C) II and IV D) II and III

C A recourse note means that the limited partner agrees to pay the note no matter what happens. He is legally liable for the $40,000, which makes both his tax basis and maximum loss potential $50,000.

Which of the following registers the securities and packages the program for a limited partnership? A) Property manager B) General partner C) Syndicator D) Limited partners

C A syndicator handles the registration of the limited partnership units.

Synapse Communication Corporation (SCC) is growing. To finance the expansion, the company has a $100 million debenture offering. Attached to the offering are five-year warrants to purchase SCC common shares. Each warrant allows for the purchase of two SCC shares at a price of $53 per share. Three years after the issue date, SCC stock is trading at $63 per share. Each warrant has A) an intrinsic value of $5. B) no intrinsic value, only time value. C) an intrinsic value of $20. D) an intrinsic value of $10.

C A warrant has intrinsic value when the exercise price is lower than the stock's current market price. In this question, each warrant allows the holder to purchase two shares of a $63 stock for $53 per share. That is a $10 per share value times two. Therefore, intrinsically, the warrant is worth $20. With two years to go, it also has time value, but the question is not dealing with that. ** This question deals with material not covered in your LEM, but it relates to recent rule changes and/or student feedback.

Which of the following statements regarding an official statement are true? It is required by the SEC for all new issues. It is required by the Municipal Securities Rulemaking Board (MSRB) for all new issues. It must be delivered to purchasers at or before settlement. It is generally used by underwriters to help sell the issue. A) I and IV B) II and III C) III and IV D) I and II

C An official statement is a document similar to a prospectus and is furnished, in most cases, to buyers of new issue municipal bonds. SEC rules require that an official statement be prepared for most—but not all—new municipal issues. The MSRB has no such requirement, as it does not regulate issuers.

In terms of the number of issues traded, the largest secondary market for securities is the over-the-counter market (OTC). Which of the following securities cannot be traded OTC? A) Preferred stock listed on the NYSE B) Exchange traded funds C) Mutual funds D) U.S. Treasury bills

C Any security that trades in the secondary markets may be traded in the OTC market. That includes securities listed on the stock exchanges. Mutual funds (and variable annuities) are securities for which there is no secondary market trading. Shares (or units) in these securities are bought and redeemed through the issuer.

A city has issued bonds to construct a new sewage treatment facility. If the bonds are not backed by the full taxing authority of the city, all of the following statements about the bond issue are true except A) the interest on these bonds is not considered a preference item for the alternative minimum tax. B) if earnings fall short of the amount needed to make principal and interest payments, the debt service reserve can be used. C) the disbursement of principal and interest payments must be approved semiannually by the state public service commission. D) there is no debt limitation on the issue.

C As an exclusion question, we are looking for the false statement. The public service commission would have no approval power over revenue bond interest and principal payments. Because the bond is not backed by the taxing authority of the city, it is a revenue bond rather than a general obligation bond. The funds for payment of interest and repayment of principal are generated through the fees paid by those using the city's water and sewage facilities. Being a public, rather than private, facility, these would not be alternative minimum tax bonds.

QED Corporation, whose common stock is currently selling for $90 per share, is having a rights offering. The terms of the offering require seven rights plus $83 to subscribe to one share of stock. Compute the theoretical value of a right on the ex-rights date. A) $0.875 B) $7.00 C) $1.00 D) $1.125

C Because this is ex-rights (without the rights), the formula does not include the "+1." The formula is (M − S) N. Plugging the numbers in, we have ($90 − $83) 7 = $7.00 7 = $1.00.

Flag Mountain Floating Rate Capital, a business development company (BDC), has the majority of its assets invested in debt securities. Income distributions are made in the form of A) interest. B) capital gains. C) dividends. D) a return of capital.

C Business development companies (BDCs) are closed-end investment companies registered under the Investment Company Act of 1940. In addition, they are regulated investment companies (RICs) under the Internal Revenue Code, meaning that BDCs must distribute at least 90% of their net investment income (NII) as dividends to shareholders. ** This question deals with material not covered in your LEM, but it relates to recent rule changes and/or student feedback.

Under SEC Rule 10b-13, a company that is the target of a tender offer must provide its shareholders with a statement indicating acceptance or rejection of the offer within how many business days of the announcement? A) 5 B) 15 C) 10 D) 20

C Once a tender offer is announced, the target company, within 10 business days of the announcement, must provide its shareholders with a statement indicating acceptance or rejection of the offer and the reasons for the position taken.

An investor has purchased a municipal certificate of participation (COP). COPs can be characterized by all of the following except A) the holder of the COP participates in lease or loan payments from a specific piece of equipment or facility purchased or built by the municipality. B) the holder of a COP could foreclose on the asset generating the revenue in the case of default. C) they would require voter approval before a municipality could issue them. D) they are a form of municipal revenue bond.

C COPs are considered revenue issues and, therefore, do not require voter approval. They are a form of lease revenue bond that allow the holders of the certificates to participate in some revenue stream (lease or loan payments) associated with land, equipment, or facilities purchased or built by the municipality. They are unique in that in the case of default, the holders of the COPs could foreclose on the asset associated with the certificate.

A direct participation program (DPP), organized as a limited partnership, must avoid at least two characteristics of a corporation. Which two characteristics are easiest to avoid? A) Centralized management and continuity of life B) Freely transferable interests and centralized management C) Continuity of life and freely transferable interests D) Continuity of life and centralized management

C Continuity of life and freely transferable interests are the easiest to avoid. The limited partnership is formed to exist for a limited time, and general partner (GP) must approve any transfer of interests. Centralized management is the hardest characteristics to avoid because management of the program is the responsibility of the general partner (GP), so management is centralized.

Covered call writing normally occurs in A) a volatile market. B) a falling market. C) a stable market. D) a rising market.

C Covered call writing normally occurs in a stable market. In a rising market, writing calls against a long stock position limits upside potential. In a falling market, the calls only provide downside protection to the extent of the premium received.

All of the following records must be retained for 3 years except A) fingerprint cards for terminated personnel. B) copies of retail communication. C) customer statements. D) audio tapes of orders handled by the trading room.

C Customer statements must be retained for 6 years. The other choices are all 3-year records.

FINRA's 5% markup policy does not apply to A) REITs. B) commissions. C) issues sold by prospectus. D) third-market trades.

C FINRA's 5% markup policy applies to all secondary market trades, whether customers are charged markups, markdowns, or commissions. Issues sold by prospectus and municipal securities, however, are exempt from the policy.

One of the key requirements in offering a DPP to a customer is that the program must be suitable. FINRA has some specific suitability requirements for DPPs. Among those is the investor A) has sufficient net worth to be deemed an accredited investor. B) does not own a DPP that will compete with the program being offered. C) has a net worth sufficient to sustain the risks of the DPP, including loss of investment. D) has sufficient experience in the type of business the program is undertaking.

C FINRA's Rule 2310 lists a few suitability standards necessary for recommending DPPs. Among those is the need for the investor to have a net worth sufficient to sustain the risks of the DPP, including loss of the investment. Although many DPPs, but not all, are limited to accredited investors, that is not a FINRA suitability standard; that is an SEC requirement. It is the general partner who cannot be in a business that competes with the DPP.

All of the following statements regarding the Federal National Mortgage Association (FNMA) are true except A) FNMA pass-through certificates are not guaranteed by the U.S. government. B) interest on FNMA certificates is taxable at all levels. C) FNMA is owned by the U.S. government. D) FNMA is a publicly held corporation.

C FNMA is a publicly held corporation. The interest income on all mortgage-backed securities is fully taxable. Though a government agency, FNMA pass-through certificates are not guaranteed by the U.S. government. The only U.S. agency whose securities are considered direct obligations of the U.S. government is the Government National Mortgage Association.

A member firm must prefile its retail communications if the member has not previously filed with FINRA. it appertains to standardized options prior to the distribution of the ODD. the advertising relates to single stock futures. under an order from FINRA to prefile. A) I and IV B) II and IV C) I, II, III, and IV D) II and III

C For a member that has not previously filed retail communications with FINRA, prefiling is required for the 1-year period following the first filing. Retail communications relating to single stock futures must be prefiled, as do options if prior to the distribution of the options disclosure document. In addition, if a member is under an order from FINRA to prefile, it must do so.

A sophisticated investor wants to purchase stock of a foreign company or an American depositary receipt (ADR) representing the shares of that company. The purchase would align with the investor's goal of growth and income, but he makes several statements about the potential purchase, and only one of them is accurate. You feel it is important to point out and discuss from a suitability perspective which statements were and were not accurate. Which of the following is the accurate statement? A) With the ADRs, I'll have voting rights just like I would if I purchased the shares directly. B) The direct purchase of the foreign stock shares eliminates currency risk. C) The purchase of ADRs representing the shares exposes me to currency risk. D) I can trade the foreign shares right here on U.S. exchanges.

C From a suitability perspective, correcting any inaccuracies about an investment that an investor might have is important. Currency risk cannot be avoided when investing in foreign companies, either directly or using ADRs. While ADRs trade on U.S. exchanges, foreign shares do not, and ADR issuers generally do not pass on voting rights to the ADR holders.

A customer buys a real estate limited partnership interest by contributing $20,000 and signing a nonrecourse note for $50,000. The customer's beginning basis is A) $20,000. B) $30,000. C) $70,000. D) $50,000.

C Generally, nonrecourse debt does not add to basis because the limited partner is not responsible (at risk) for the repayment of the debt. However, in real estate partnerships, the at-risk rules do not apply, and therefore, add to basis in this type of partnership.

If a member wishes to appeal an adverse decision in a Code of Procedure hearing, the member first must appeal to the National Adjudicatory Council within how many days of the decision date? A) 30 B) 45 C) 25 D) 40

C If either side is displeased with a Code of Procedure decision, an appeal must be made within 25 days of the decision date.

IBM sold computers to a Soho retailer and agreed to accept payment of 10 million British pounds in 65 days. In which of the following ways could the company protect the payment against adverse foreign currency fluctuations? A) Buy U.S. dollar puts B) Buy pound calls C) Buy pound puts D) Buy U.S. dollar calls

C If the company wants to protect its investment, it has to protect against the payment going down in value relative to the cost. To protect against the value of the payment going down in relation to the cost, the company would buy puts on the payment currency. To offset the cost of the puts, the company will also sell calls. The calls will be covered by the ownership of the actual currency. Remember the acronym EPIC: Exporters buy Puts and Importers buy Calls.

Compared with selling short, it would not be correct to state that buying a put option A) requires a smaller capital commitment. B) does not require meeting the locate requirement for short sales. C) offers a high potential profit. D) has a lower loss potential.

C In both cases, the investor profits when the price of the underlying asset declines. The maximum profit is when the price of the asset reaches zero. That makes the maximum profit on the long put equal to the strike price minus the premium. Because there is no cost in selling short (the margin deposit requires a capital commitment, but is the investor's money), the maximum profit is the entire decline to zero. For exam purposes, we disregard commissions. Buying a put requires a smaller capital commitment than does shorting the stock and has a lower loss potential (the premium only) because selling short involves unlimited risk. When selling stock short on an exchange, the shares to be borrowed must be located before the sale. This is not a requirement when buying a put.

Debt service on an industrial revenue bond is secured by A) sales taxes. B) special assessments. C) lease payments paid by a corporation. D) ad valorem taxes.

C Industrial revenue bonds are issued by a municipality or an authority established by a municipality. No municipal assets or general revenues are pledged to secure the issue. The net lease payments by the corporate user of the facility are the only source of revenue for debt service.

Last year, ABC Mutual Fund paid dividends of $1.50 per share and distributed $0.80 per share in capital gains. The fund has a bid price of $13.50 and an asked price of $14.20. An investor who purchased shares in this fund nine months before the distributions receives $100 in nonqualifed dividends and $53 in capital gains. All distributions were reinvested in additional shares. If this is the individual's only investment, she will not be required to pay federal taxes on the dividend income. be required to pay federal taxes at the ordinary income rate on the dividend income. be required to pay capital gains tax on $53. not be required to pay capital gains tax on $53. A) I and IV B) I and III C) II and III D) II and IV

C Investors are required to pay taxes on all distributions from mutual funds; it makes no difference if they are taken in cash or reinvested in additional shares. Nonqualified dividends are taxed at ordinary income rates. Capital gains distributions are always considered long term and taxed at the capital gains rate, unless something in the question states otherwise.

When acting as an agent for a customer, Municipal Securities Rulemaking Board (MSRB) rules require the broker to make a reasonable effort to obtain which of the following? A fair price in relation to prevailing market conditions The best price A reasonable price in relation to prevailing market conditions Quotes from at least three municipal dealers or one broker's broker A) II and III B) II and IV C) I and III D) I and IV

C MSRB rules require only that municipal securities dealers effect trades for customers at prices that are fair and reasonable in current market conditions.

If an investor is in the highest federal income tax bracket and is subject to the alternative minimum tax (AMT), which of the following securities should an agent recommend? A) Treasury bond B) Corporate bond C) General obligation (GO) bond D) Industrial revenue bond

C Municipal bonds are suitable for the portfolio of an investor who is in a high tax bracket because the interest is exempt from federal income tax. A GO bond is a better recommendation than an industrial revenue bond because the interest on industrial revenue bonds is likely subject to the AMT.

Which of the following regarding a municipal bond broker's broker are true? Protects customer identity Must disclose the identity of customers Has no inventory Maintains an inventory A) I and IV B) II and III C) I and III D) II and IV

C Municipal brokers' brokers generally purchase and sell securities on an anonymous basis for institutional clients. They are not in the business of making a market; therefore, they maintain no inventory.

New issues of municipal bonds are exempt from all of the following except A) Securities and Futures Authority (SFA) requirements. B) Securities Act of 1933 registration requirements. C) Securities Exchange Act of 1934 antifraud provisions. D) U.S. state registration requirements.

C Municipal securities are exempt from federal and state registration. However, no security is exempt from the antifraud provisions of federal securities law, including the Securities Exchange Act of 1934. The SFA is a UK regulator and has no application in the United States.

A customer is considering adding a real estate investment trust (REIT) to her portfolio. She lists all of the following as advantages. You correct your customer and point out that one of them is not an advantage of investing in REITs. Which of the following is not an advantage of investing in REITs? A) Being able to divest of the shares easily B) Using real estate as a potential hedge against the movement of other equity securities the customer owns C) Dividend treatment D) Having a professionally managed portfolio of commercial real estate assets

C Of those listed, only dividend treatment can be identified as not being an advantage. While the expectation of receiving dividends is inherently good, dividends paid by REITs to their shareholders are not recognized as qualified and are therefore taxable to the investor at their full ordinary income tax rate. The shares are traded on exchanges or over the counter and are considered liquid, and having professionally managed assets should be a plus. While real estate valuation and price movements are subject to many forces, historically, real estate has provided some hedge against the movements of other equity securities.

Which of the following mutual funds should an investment adviser representative recommend to a corporate client whose objective is current income with moderate risk? A) Aggressive growth fund B) High-yield bond fund C) Preferred stock fund D) Money market fund

C Preferred stock generates current income in the form of dividends. Aggressive growth funds strive for capital appreciation rather than current income. Money market funds have low yields, not the high yields that an income investor wants. While high-yield bonds provide current income, they entail a high—rather than moderate—degree of risk.

An investor engages in a wash sale when he repurchases the same security or purchases a substantially identical security of the issuer within 30-days of a sale resulting in a loss. If an issuer's common stock were sold at a loss, all of the following would be considered substantially identical except A) call options. B) convertible bonds. C) preferred stock. D) warrants.

C Preferred stock is not substantially identical because it cannot be converted or exchanged into the common shares that were sold. All the other securities could be converted or exchanged into the common stock sold. That makes them considered substantially identical. Please note: Preferred stock is not convertible unless something in the question says it is.

The Securities Exchange Act of 1934 deals with all of the following except A) monitoring accounts for insider trading violations. B) marking sales long or short on an order ticket. C) filing an updated prospectus. D) filing of financial statements by broker-dealers.

C Prospectus filing is a requirement of the Securities Act of 1933.

A registered principal must approve all orders A) prior to entry. B) prior to execution. C) by the end of the trading day. D) within one business day.

C Registered principals must approve all orders promptly after execution. FINRA interprets this term, however, as by the end of the trading day. Orders may always be approved earlier, but are not required to be.

A senior official of an issuer learns that nonpublic information was disclosed to an institutional shareholder of the issuer and/or an analyst covering the issuer at a private meeting. To avoid violating Regulation FD, the issuer must do which of the following? A) Promptly disclose the information as soon as reasonably practical, but in no event after the later of 24 hours or the start of the next day's trading on the marketplace where that security trades. B) Nothing need be done as long as no trading is done based on the selective disclosure. C) Promptly disclose the information as soon as reasonably practical, but in no event after the later of 24 hours or the start of the next day's trading on the New York Stock Exchange. D) Disclose the selective disclosure in its next SEC filing.

C Regulation FD prohibits selective disclosure to analysts and institutional shareholders. The disclosures must be made to all shareholders of the issuer. The SEC has agreed that public conference calls, press releases or press conferences, and webcasts are FD-compliant methods of public disclosure. Please note that the alternative of the start of the next day's trading is based on the NYSE regardless of where the security is traded.

In general, investors pay a commission rather than a sales charge when purchasing shares of A) a front-end load fund. B) an open-end investment company. C) a closed-end investment company. D) a mutual fund.

C Sales charges could be paid on all types of open-end funds. Commissions are paid on securities traded in the secondary market, such as closed-end investment company shares. On the exam, open-end investment company will always refer to a mutual fund unless the question is about the structure of an exchange-traded fund (ETF).

Many parents find that opening an UTMA account for their child is not only a good way to accumulate funds for the future but also a good way for the child to gain an appreciation for investing. The account custodian may use principal as well as income generated in the account to pay for all of the following for the child except A) summer camp. B) private school. C) new clothes. D) the latest model smartphone.

C The Uniform Transfer to Minors Act permits the custodian to use the funds for almost anything that is a benefit to the minor. The primary exceptions are those that most states consider to be the parental obligations of food, clothing, and shelter.

If a limited partnership interest is sold, the gain or loss in the sale is the difference between the sales proceeds and A) the original basis. B) the total of tax preference items allocated to the investor. C) the adjusted basis. D) the total of the deductible losses taken by the investor.

C The adjusted basis is a limited partner's cost basis at any point in time. Gain or loss on the sale of the partnership is determined by comparing the sales proceeds to the adjusted basis.

A customer requests that his broker-dealer hold his fully paid for stock. Which of the following are required? A written stock power from the customer Full power of attorney from the customer to the broker-dealer The securities must be segregated from those of the firm and other customers The customer must be informed that the securities may be withdrawn by him at any time A) I and II B) II and IV C) III and IV D) II and III

C The broker-dealer must segregate the customer's fully paid for securities and inform the customer that the securities can be withdrawn at any time.

Your customer is opening a new options account. Which of the following need not occur to open the account? A) The registered representative must document that the client has received a current OCC disclosure document. B) The client must agree that any material change in financial status requires the broker-dealer be notified and the options agreement be amended. C) The OCC must verify the financial information supplied by the client to ultimately approve the account. D) The background and financial information provided by the client must be verified by the client and returned within 15 days of the time the account was approved.

C The client must have a current Options Clearing Corporation (OCC) disclosure document. This is verified by the client's signature on the options agreement form, which must be signed and returned within 15 days of account approval. The client must agree to notify the firm of any changes in financial status as soon as possible, and the options agreement must be amended, if necessary. OCC approval for an options account is not required, nor do they verify any information given by the client.

In a variable life annuity with 10-year period certain, a contract holder receives A) fixed payments for 10 years, followed by variable payments for life. B) 10 years of variable payments. C) a minimum of 10 years of variable payments, followed by additional variable payments for life. D) variable payments for 10 years, followed by fixed payments for life.

C The owner of a life annuity with 10-year period certain will receive payments for life, subject to a minimum of 10 years. If the contract holder dies before the period expires, the remaining payments are made to the beneficiary. For example, if a life annuity with a 10-year period certain contract holder died after five years, payments would continue for five more years to the beneficiary and then stop.

American depositary receipt (ADR) owners have all the following rights except A) the right to receive dividends in U.S. dollars. B) the right to receive the underlying foreign security. C) the right to sell the ADR in the foreign market. D) the right to sell in the secondary market.

C The purpose of the ADR is to facilitate trading in U.S. markets. The ADR can only be traded in the United States. If the owner exercises the right to obtain the actual foreign security, it may be sold overseas.

A producer of fine French wines has just signed a contract to export $10 million of wine to a distributor in the United States. Using listed foreign currency options, this producer would have the best protection against currency risk by A) taking a long position in U.S. dollar puts. B) taking a long position in U.S. dollar calls. C) taking a long position in euro calls. D) taking a long position in euro puts.

C There are no listed options in the U.S. dollar. That reduces your choice to a long euro put or call. Because the contract will be paid for in dollars, the producer is concerned that the dollar will fall against the euro. Or, stated another way, the concern is that the euro will rise against the dollar so that the $10 million will not buy as many euros as on the day the contract was signed. When one is afraid the price of asset will rise, such as those who take a short position in a stock, the best protection is buying a call. The easiest way to remember this is through the acronym, IPEC - Importers buy Puts and Exporters buy Calls. This is used when the party involved in the question is in a foreign country. Because we are dealing with a French exporter, buying calls on the local currency offers the best protection.

If an indenture has a closed-end provision, this means A) a sinking or surplus fund must be established. B) the bonds must be called before maturity. C) additional issues will have junior liens. D) additional issues have no lien on the revenue stream.

C These additional issues are also known as junior lien bonds. Under a closed-end indenture, additional bonds issued against the same stream of revenues have a junior (subordinate) claim to those already outstanding unless the funds are required to complete construction of the facility.

An investor purchased 100 shares of Paradigm Publishing Corporation (PPC) on October 17, 2020. The price was $83 per share. On April 11, 2021, the investor wrote one PPC Nov 85 call for 3. At expiration date, the PPC stock is selling for $80 per share, and the investor liquidates the stock at the market price and the option at its intrinsic value. The net tax consequences are A) $300 long-term loss. B) $300 long-term gain. C) no gain, no loss. D) $200 short-term gain.

C This is a long stock/short call position, better known as a covered call. Liquidating the stock at the market value of $80 per share results in a $300 long-term loss ($83 minus $80, held for 13 months). With the market price at $80, the 85 call expires worthless resulting in a $300 short-term gain (the entire $300 premium). The question asks for the net tax consequences. The $300 gain on the option is offset by the $300 loss on the stock (there is no intrinsic value to an 85 call when the market is 80). One could also arrive at the correct answer by computing the breakeven point (the $83 purchase price minus the $3 premium received) of $80. Because the stock was sold at the breakeven point, there is no loss or gain.

One of your customers is interested in purchasing the shares of a new issue from a local manufacturing company. The issue is for $15 million of common stock. The investor's net worth is $95,000 and net income is $75,000 per year. The plan is to invest $15,000 into this stock. Under Regulation A of the Securities Act of 1933, this investment is A) not permitted because Regulation A requires investors to be accredited. B) not permitted because it represents more than 10% of the customer's net income. C) permitted. D) not permitted because it represents more than 10% of the customer's net worth.

C Under Regulation A, an offering of $20 million or less in a 12-month period is a Tier 1 offering. Unlike a Tier 2 (up to $75 million) offering, there are no restrictions based on net worth or net income. A separate question, but not relevant to this one, is the suitability of this investment. It is important to stick with the question being asked.

At 3:55 pm ET, a registered representative receives a market order from an officer of XYZ to buy 75,000 shares of XYZ for the company's account. The registered representative must A) advise the officer that a safe harbor under SEC Rule 10b-18 no longer exists before refusing the order. B) place the order without taking any further action. C) advise the officer that a safe harbor under SEC Rule 10b-18 no longer exists before placing the order. D) refuse the order.

C Under SEC 10b-18, an issuer purchasing its own securities cannot affect the opening or closing of the security. A safe harbor is available if the issuer is not involved in the first transaction of the day or in any transaction in the last 30 minutes of trading (10 minutes if the security is actively traded). Compliance with the rule is voluntary. If an issuer wants to reduce or eliminate its regulatory liability, it must satisfy the safe harbor conditions. Otherwise, repurchases will not fall under the safe harbor for that day. If the issuer were to purchase its own securities during the last 30 minutes of trading, it may be forced to justify that its purchase did not affect the closing price. If a registered representative receives an order from an issuer at 3:55 pm ET, he must advise the issuer that a safe harbor is not available. The representative may then place the order if instructed to do so.

All of the following statements regarding Section 529 plans are true except A) states impose very high overall contribution limits. B) contributions to a 529 plan may be subject to gift taxation. C) the income level of the contributor can affect the annual contribution amount. D) the assets in the account are controlled by the account owner, not the child.

C Unlike Coverdell ESAs, the income level of the contributor will not affect annual contributions under a Section 529 plan.

If a customer buys $28,000 of ABC stock in April 20XXand at year end, the stock is worth $23,000, how much may the customer deduct on his 20XX tax return? A) $5,000 B) $3,000 C) $0 D) $2,000

C Until the customer realizes the loss by selling, there is no tax deduction

Parker has been a client of Enigma Mathematical Portfolio Modeling (EMPM), a FINRA member broker-dealer, for 10 years. Parker has decided that it is time to move the account to a new firm, Turing Technical Analytics, (TTA). Which of the following statements accurately reflects the requirements when using the ACATS system? A) The transfer initiation form (TIF) is sent to ACATS by the carrying firm. B) The transfer initiation form (TIF) is sent to ACATS by the customer. C) The transfer initiation form (TIF) is sent to ACATS by the receiving firm. D) The transfer initiation form (TIF) is sent to the carrying firm by the receiving firm.

C When a customer wishes to transfer an account from one member firm to another, the customer's signature is required on the TIF. That TIF is then sent to ACATS by the receiving firm (the one who will be getting the new account—Turing Technical Analytics (TTA).

If a customer gives his broker-dealer an order to sell his stock if it falls to or below 69 and will not accept a price below 69, the order is A) a sell limit order. B) a buy limit order. C) a stop limit order. D) a stop order.

C When an order is entered this way, the client has specified that it should not be triggered until the stock is at or below 69, a stop order. Because the client will not accept an execution below 69, it is a stop limit order.

In a restricted margin account, a sale of $5,000 of stock will create A) SMA of $5,000. B) buying power of $10,000. C) SMA of $2,500. D) a $5,000 decrease in the debit balance.

C When securities are sold in a restricted margin account, half the proceeds go to reduce the debit and the other half are journaled into SMA. Therefore, a $5,000 sale will generate SMA of 50% of the sale, or $2,500. Buying power is the SMA × 2, or $5,000.

Lindsey Wolfe, a public school teacher, has been contributing to a 403(b) TSA plan for the past 20 years. Contributions total $50,000 and the current value is $200,000. Wolfe is still teaching full time for the school system. When does Wolfe have to begin taking required minimum distributions? A) Required minimum distributions are never required for annuities B) At age 59½ C) At age 72 or when no longer working for the school system, whichever is later D) At age 72

C Wolfe is invested in a qualified annuity. Therefore, the minimum distribution requirements are the same as for any qualified account. RMDs must begin at age 72 but can be postponed as long as continuously employed by the same employer. Unless qualifying for an exception, any withdrawals from a qualified annuity before reaching age 59½ are taxed as ordinary income with the additional 10% penalty.

Yield-based options expire on A) the day following the third Friday of the month. B) the third Wednesday of the month. C) the last day of trading. D) the day after the last day of trading.

C Yield-based options expire like stock options—on the third Friday of the expiration month, which is the last day of trading.

An analyst is comparing the yields of U.S. Treasury bonds and AAA-rated corporate bonds with similar maturities. This measurement would indicate an improving economy when A) the yield spread is widening. B) the yields on the Treasury bonds are lower than those on the corporate bonds. C) the yield spread is narrowing. D) the yield spread is remaining stable.

C You should understand that the greater the risk, the higher the yield on the bond. Many analysts compare the difference between yields on bonds with the same maturity but different quality (rating) to get a sense of the market sentiment. One common measurement is the difference in yields between Treasuries and corporate bonds. This difference is called the yield or credit spread and tends to widen when economic conditions sour and narrow when they get better. You will never see (at least on the exam and probably in the real world as well) the yield on a corporate bond lower than a Treasury bond when the maturities are similar. It is the spread, the difference in yields, that is important. ** This question deals with material not covered in your LEM, but it relates to recent rule changes and/or student feedback.

T-bills are quoted A) in 16ths. B) in 32nds. C) as a percentage of par. D) on an annualized discount yield basis.

D T-bills do not bear interest. T-bills trade and are quoted on an annualized discount yield basis.

Which of the following securities can generate phantom income? A) Treasury bills B) Treasury notes C) Treasury bonds D) TIPS bonds

D TIPS bonds adjust the principal value each six months based on the inflation rate. If the inflation rate is positive, the value increases. Those increases are reported as income each year even though the investor does not receive the appreciation until the bonds mature (or are sold).

In a competitive bid, which of the following would the issuer need to determine net interest cost? Coupon rates Basis Dollar price Spread A) II and IV B) I and II C) III and IV D) I and III

D The coupon rate and dollar price are important to the issuer because they determine the actual cost of borrowing. The spread and basis at which the bonds will be resold are important to the underwriters but not to the issuer.

A J&J Treasury bond with a 5% coupon having a maturity date of July 1, 2039, is purchased in a transaction for cash on February 24. What is the number of days of accrued interest shown on the trade confirmation? A) 55 B) 63 C) 53 D) 54

D A bond begins accruing interest on the prior interest payment date (January 1) and accrues up to, but not including, the settlement date (February 24). Did you notice that this was a transaction for cash? That means the settlement date is the same day as the trade (February 24). Normally, Treasury securities settle T+1. If this was a regular-way trade, the accrued interest would be 55 days because settlement would have been February 25. Be careful reading the question; it is easy to skip over critical information. Because accrued interest on government bonds is computed actual days, actual year, 31 days for January plus 23 days for February, it equals 54 days.

An investor who is bearish on the outlook for Fernweh Travel Services (FTS) sells 100 shares short at $52 per share. Three months later, the market price of FTS shares is $58. Under FINRA rules, a maintenance call will be issued when the per share price of FTS A) increases by more than $9.60. B) increases by more than $17.40. C) decreases by more than $2. D) increases by more than $2.

D A short margin account reaches the maintenance level when the equity in the account reaches 30% of the market value of the short stock. To find that level, divide the credit balance by 130% (or 1.3). The credit balance is the sum of the sale proceeds plus the Regulation T initial margin requirement. In our question, sale proceeds are $5,200 ($100 shares times $52 per share). To that we add the 50% Regulation T requirement ($2,600) resulting in a credit balance of $7,800. Dividing that $7,800 by 1.3 = $6,000. That tells us that the highest the price of FTS shares can be is $60 per share. Anything above that will trigger the maintenance call. With the current market price of $58, anything in excess of a $2 per share increase will result in a maintenance call. Remember, when an investor sells short, losses occur as the market price rises.

Which of the following would have the least market risk? A) Fannie Maes B) Corporate or municipal bonds with long-term maturities C) AAA corporate debentures D) Revenue anticipation notes

D Anticipation notes are the shortest term, which gives them the least market risk (the risk that price will fluctuate during the time left to maturity).

If a registered options principal is asked to approve a discretionary order to buy 1 XYZ Oct 60 put and sell 1 XYZ Oct 55 put for a net debit of $5, he should A) approve the order if the customer has sufficient funds in her accounts. B) approve the order in writing. C) obtain the best execution for the order. D) not approve the order.

D Because this is a debit spread, the maximum gain occurs if both sides are exercised. If this occurs, the investor earns $5 (buy stock at 55 when the short put is exercised and sell stock at 60 by exercising the long put). Because the net premium paid for the spread is $5, there can never be any gain. This spread is not economical.

One of your customers calls you to say that he received a letter saying that his local water works revenue bonds were being defeased. How would that affect the customer? A) The maturity date is being automatically extended as called for in the official statement. B) The customer will need to file a claim with the appropriate court to receive payment for the bonds. C) Because of failure to generate sufficient revenue, interest payments are suspended temporarily. D) The customer would be receiving payment of the principal plus any accrued interest after the defeasance is completed.

D Defeasance occurs when an outstanding bond issue is paid off prior to maturity through a refunding. Once the creditors (the bondholders) have received their funds, any liens on assets or revenues are terminated. **This question deals with material not covered in your LEM, but it relates to recent rule changes and/or student feedback.

Dale Johnston has been a registered representative with Consolidated Investment Services (CIS) for over 20 years. Taylor Kahn has been Johnston's client for most of that time. Kahn recently reached full retirement age for Social Security and has begun using those funds for investment. Johnston's practice is to speak with clients on a quarterly basis, unless something merits a special call. It seems to Johnston that on the last call, Kahn seemed a bit confused over the strategy being used with the Social Security funds. A second call seemed to verify that Kahn was still a bit fuzzy. This morning, a phone call came in from Kahn's son asking Johnston to sell one of the holdings and forward the proceeds to his bank. Kahn's son does have a full power of attorney over the account but has never given instructions to have money sent to him before. What is the most appropriate action for Johnston to take? A) Refuse to complete the trade B) Place the trade after receiving approval from your supervising principal C) Freeze the account D) Place a temporary hold on the withdrawal

D Effective March 17, 2022, amendments to FINRA Rule 2165 on senior exploitation changed the maximum length of the hold and extended the ability to hold any transaction, not just disbursements. The previous maximum length of 25 business days (15 automatic and an additional 10 if justified) has now been extended to 55 business days if the member firm has reported the matter to a state regulator or agency or a court of competent jurisdiction.

One of your customers owns a single premium deferred variable annuity policy. The customer has been discussing a policy offered by a different insurance company that has a wider selection of investment portfolios. Should the customer elect to engage in a Section 1035 policy exchange, A) principal approval of any sale or exchange (including a 1035 exchange) must be obtained within three business days. B) you need to explain that any earnings on the exchanged policy will be subject to tax at ordinary income rates plus a 10% penalty if the customer is not at least 59 ½. C) the customer will not be able to add any additional funds to the new policy. D) principal approval of any sale or exchange (including a 1035 exchange) must be obtained within seven business days.

D FINRA Rule 2330 requires a registered principal to review and determine whether to approve a customer's application for a deferred variable annuity before sending the application to the issuing insurance company. This must occur no later than seven business days after the broker-dealer receives a complete and correct application. It makes no difference if this is an initial sale or, as is the case in this question, an exchange (which is a purchase of a new policy using the proceeds of an old one). When done properly, there are no taxes or penalties with a Section 1035 exchange. Even though the old policy was purchased in a single premium, there is generally no problem with adding money when making an exchange. ** This question deals with material not covered in your LEM, but it relates to recent rule changes and/or student feedback.

Which of the following statements regarding the Government National Mortgage Association (GNMA) is true? A) GNMA originates loans to home buyers and sells the mortgage-backed securities to private lending institutions. B) Lending institutions apply to GNMA for funds to lend to residential home buyers. C) GNMA approves residential mortgages for home buyers. D) Private lending institutions approved by GNMA originate eligible loans and sell the mortgage-backed securities to investors.

D GNMA is a government-owned corporation that approves private lending institutions, such as banks and mortgage companies, to originate eligible loans, pool them into securities, and sell the GNMA mortgage-backed securities to investors. GNMA does not originate loans, and it does not issue or sell securities.

All of the following about a good faith deposit on a municipal bond underwriting are correct except A) it is applied against winning bidder's payment for the issue. B) it is lost if the syndicate fails to carry out the provisions of the underwriting agreement. C) it is returned to the unsuccessful bidders. D) it is returned to the winning syndicate to cover any loss incurred in the subsequent sale of the bonds.

D In a municipal bond underwriting, the good faith deposit is submitted by a potential syndicate as earnest money. If the syndicate is not awarded the issue, the check is returned. If the syndicate is awarded the issue, the money is applied against the payment. However, if the syndicate fails to carry out the provisions of the underwriting, the money is retained by the issuer. If you have ever bought or sold a house, this is comparable to the earnest money deposit turned in by the buyer with the offer.

Under FINRA rules, members are prohibited from soliciting votes from limited partners in connection with a proposed rollup unless any compensation to be received by the member A) does not exceed 5% of the value of the securities to be received in the exchange. B) does not exceed 15% of the value of the securities to be received in the exchange. C) does not exceed 10% of the value of the securities to be received in the exchange. D) does not exceed 2% of the value of the securities to be received in the exchange.

D In connection with a DPP rollup, member firms may not solicit votes from limited partners unless the compensation is 2% or less. The 10% limitation is the maximum compensation in the sale of a DPP. The 15% limitation is the maximum percentage of the gross proceeds of a DPP that may be used for the organization and offering expenses. The 5% is likely an attempt to make you think about the FINRA 5% markup policy. That does not apply to DPPs.

Recent years have shown an enormous growth in the sales of exchange-traded funds (ETFs). Some of the benefits of using ETFs in your clients' portfolio would include A) greater management flexibility than most comparable mutual funds. B) greater diversification than most comparable mutual funds. C) lower risk than most comparable mutual funds. D) greater tax efficiency than most comparable mutual funds.

D In general, largely because of the lack of need to rebalance the portfolio to meet investor redemption requests or market changes, ETFs have fewer taxable events. That translates into fewer taxes to their shareholders. Because an ETF is generally restricted to those securities in the index it tracks, there will be the same diversification as a mutual fund tracking that same index. Because the mutual fund is not necessarily tied to an index, the greater management flexibility can offer greater diversification. The risk is based on the portfolio, not the structure of the company.

If a customer wishes to change a day order to a good-til-canceled (GTC) order in the middle of the day, the registered representative should A) enter the new GTC order immediately and do nothing about the day order. B) enter a change notice immediately. C) enter the new order as GTC and immediately cancel the day order. D) allow the day order to expire at the end of the day and put in the GTC order before the next day's opening.

D The GTC is treated as a new order. The registered representative should wait until the close of trading so as not to lose the time priority of the original order that day.

Gerry Logan has been managing his own securities portfolio for the past 15 years. His returns have been about the same as the S&P 500, and as he gets older, he does not want to have to spend the time and effort to keep up the performance. Logan is currently 55 years old and has sufficient discretionary income and savings that enable him to take moderate risks. He is of the belief that his own experience proves that you can't beat the market. Which of the following would you suggest for him? A) 50% into an S&P 500 index fund (or ETF), 50% into an investment-grade bond fund B) 50% into an S&P 500 index fund (or ETF), 50% into a money market mutual fund C) 100% into an S&P 500 index fund (or ETF) D) 50% into an S&P 500 index fund (or ETF), 30% into an international index fund and 20% into an investment-grade bond fund

D Index funds (or ETFs) are appropriate investment vehicles for investors who believe that active management does not produce returns above the cost. Investing in the S&P 500 would give him returns comparable to what he has enjoyed in the past. However, in recognition of his advancing age, it would appear prudent to diversify by placing some of the money into the international sphere and a portion into the safety of high-grade debt securities.

Which of the following transactions would not be subject to the 5% markup policy? A) Your client enters trades to purchase two different mutual funds in the same fund family. The combined purchases do not qualify for any breakpoints. The client is charged a sales charge of 6.5%. B) A client sells shares of an over-the-counter stock and uses the proceeds to purchase shares from your firm's inventory account. C) Your firm agrees to do an agency cross-transaction between two of your clients. Each client has been charged a commission. D) A client enters an order to purchase one share of a stock to be put in the name of her grandchild. You charge the client the minimum commission for your firm ($15) even though the stock is currently trading at $26 per share.

D It is important to understand that the FINRA Markup Policy is a guide, not a rule. The policy describes the various factors that a member firm should consider when determining the markup, markdown, or commission to charge in any given transaction. Because securities sold by prospectus have a stated offering price per share, a guide is unnecessary; the price is the price. In the case of mutual funds, which are always sold by prospectus, the formula for computing the price (NAV plus the sales charge, if any) is clearly listed in that prospectus. Once again, member firms do not need a guide; the price is the price. The policy is frequently referred to as the five percent policy, but that number is simply a guideline, and the broker-dealer's charge can exceed that percentage, especially in the case of a small transaction, such as one share at $26.

A pension plan might invest in each of the following except A) corporate bonds. B) equities. C) variable annuities. D) tax-free municipal bonds.

D It is inappropriate to place tax-free investments into a tax-deferred plan because there is no benefit to the deferral.

In recent years, much publicity has surrounded the rapid growth of start-up businesses. In most cases, the early financing was done privately. When private debt is used at the intermediate stage of a company's development, it is called A) mid-term debt. B) intermediate debt. C) middle-risk debt. D) mezzanine debt.

D Just as the mezzanine in a theater is between the balcony and the orchestra levels, mezzanine debt represents financing supplied at the intermediate point in a new company's development. The funds are provided on a private basis and the investment carries a high degree of risk. As an alternative investment, it will be suitable for a very narrow range of customers. **This question deals with material not covered in your LEM, but it relates to recent rule changes and/or student feedback.

Bond trust indentures are required for A) Treasury securities. B) municipal revenue bonds. C) municipal general obligation bonds. D) corporate debt securities.

D Municipal and government bonds are exempt from the trust indenture requirement of the Trust Indenture Act of 1939. Revenue bonds are frequently issued with a trust indenture, but no legal requirement to do so exists. The Trust Indenture Act of 1939 requires that corporate bond issues of $50 million or more sold interstate must be issued with a trust indenture.

A municipal issuer's net total debt is made up of which of the following? Direct debt Defeased debt Overlapping debt Paid-up debt A) I and II B) III and IV C) II and III D) I and III

D Municipal bond analysts examine an issuer's net total debt to determine an issue's safety. Debt that is not a burden on the municipality is not included in total debt. Defeased debt and paid-up debt can be removed from the municipality's debt accounts. Direct debt is debt issued by the municipality, and overlapping debt is the municipality's share of debt issued by authorities that draw revenues from the same sources as the municipality. Together, direct debt and overlapping debt constitute the municipality's net total debt.

All of the following activities in a customer's mutual fund account may be considered a violation of the Conduct Rules except A) switching Class A shares between fund families. B) short-term trading in mutual fund shares. C) excessive activity in the customer's account. D) granting discretionary authority to a new registered representative.

D Mutual funds are considered a long-term investment. Thus, switching Class A shares of funds, short-term trading of funds, and excessive activity in a customer's account very likely indicate that the registered representative is churning. There is nothing unlawful about granting discretionary authority to a new registered representative.

Treasury STRIPS and Treasury receipts are quoted based on A) amortization of premiums. B) 0.125 (⅛ of a point in dollars). C) 0.03125 (1/32 of a point in dollars). D) yield to maturity.

D Noninterest-bearing securities, like zeroes, are quoted based on their yield to maturity. They are sold at a discount and mature at par.

Your customer's portfolio consists of 40% long-term government bonds, 20% preferred stock, and 40% common shares of utility companies. Which of the following may have the single largest impact on the entire portfolio? A) Corporate earnings B) Oil and gas price movements C) Foreign currency fluctuations D) Interest rate movements

D Of the four answer choices, interest rate movement is the most likely to impact each of the portfolio components. Interest rates and bond prices have an inverse relationship, and their movement often determines whether investors might seek out investment alternatives with higher returns, such as dividend-paying utilities and fixed-dividend preferred shares.

Your customer notices that the exchange rate for the British pound in the spot market is listed at 148.47. What do you tell her when she asks you what this means? A) $1 equals 1.4847 pounds. B) $1 equals 14.847 pounds. C) One pound equals 14.847 cents. D) One pound equals $1.4847.

D The exchange rate refers to cents per British pound; 148.47 equals $1.4847.

What action could a corporation take that would result in the forced conversion of an outstanding convertible debt security? A) Reduce the coupon rate below the dividend rate on the common stock B) Exercise the conversion feature when the debt security's conversion value exceeds the call price C) Reduce the dividends on the common stock to a rate lower than the interest on the debt security D) Exercise the call feature when the debt security's conversion value exceeds the call price

D One of the investor benefits of a convertible security is that an increase in the market price of the underlying common stock will lead to a comparable increase in the price of the convertible. For example, when the market price of the common stock is $25 per share, a $1,000 convertible debenture with a conversion ratio of 50 shares per bond has a conversion value of $1,250 (50 shares time $25 per share). Because most convertibles are also callable, by calling the bonds at the stated call price (perhaps 102 or 103) the company can force the bond holders to convert the bonds. Using our example, why would investors hold onto the bonds knowing that, within about 30 days, they're going to get a check for $1,020 or so for each bond when they could convert and own shares worth $1,250 per bond. This is known as forced conversion. The coupon rate is fixed, and an investor would not want to convert to the stock just because the dividends on the stock are lower than the interest on the bond. ** This question deals with material not covered in your LEM, but it relates to recent rule changes and/or student feedback.

Which of the following statements about municipal original issue premium bonds are true? The original issue premium must be amortized. If the bond is held to maturity, there will be no capital loss reportable. The cost basis of the bond is adjusted downward by the amortized amount. A) I and III B) II only C) I and II D) I, II, and III

D Original issue premium municipal bonds (as well as those purchased in the secondary market) must be amortized by an amount each year so that, if held to maturity, there is no reported capital loss.

Popular Investment Securities, a FINRA member firm, produces short videos describing the general characteristics of different types of securities. Periodically, an interstitial appears during the video. Under FINRA's rules on communications with the public, A) interstitials may not be used in public communication without the consent of the viewer. B) video presentations of any kind must be filed with FINRA within 10 days after first use. C) as long as the presentation is strictly generic, filing with FINRA is not required. D) the appearance of the interstitial defines the video as retail communication requiring filing.

D Probably the best example of an interstitial is the pop-up ad. Sometimes it is a full-page ad causing the viewer to see the advertisement before being able to see the rest of the content. Without the interstitial, a generic video describing general characteristics of a type of security would not require filing. But, once that advertisement pops up, it is now retail communication and must be filed.

Reggie owns a convertible bond that converts into 20 shares of common stock. The current market value of the bond was 118½ at the close on Friday, April 1. A 30-day call is announced before the opening on Monday, April 4, at a price of 102. The stock is trading at $57.75. What should Reggie do? A) Hold the bond to maturity B) Redeem the bond at the call price C) Sell the bond D) Convert the bond into the stock

D Reggie will not be allowed to hold the bond to maturity because it is being called. The real question is whether he should sell the bond, allow it to be called, or convert it to the underlying stock. Now that the call has been announced, the market value of the bond will fall to meet the call price. This occurs as a result of declining demand. (Who wants to buy a bond that is about to be called at a lower price?) Thus, redeeming the bond at the call price and selling the bond would both yield the same results: $1,000 times 102% equals $1,020. If he converts the bond, he will get the following results: 20 shares times $57.75 equals $1,155. Therefore, it makes the most sense to convert the bond.

The child of one of your recently deceased clients comes to your office with several properly signed stock certificates inherited from a parent. The child does not have an account and wishes to sell the securities. An account is opened for the purpose of the liquidation. Regulation S-P would refer to this child as A) a beneficiary. B) a covered person. C) a customer. D) a consumer.

D Regulation S-P makes a distinction between consumers and customers. That is important because it makes a difference when it comes to annual reporting. A consumer is basically a "one-shot" client, as in this case. After the liquidation, this account will be closed and you probably won't ever hear from the child again. A customer has an ongoing relationship and requires annual privacy notices—the consumer does not.

When analyzing a direct participation program investment, a method that takes into account the revenues and expenses is A) liquidity analysis. B) fundamental analysis. C) technical analysis. D) cash flow analysis.

D Revenues and expenses are cash items and would be analyzed by cash flow analysis.

If a customer has a long margin account with a market value of $12,000, a debit balance of $8,000, and special memorandum account (SMA) of $2,000, how much can the customer withdraw from the account? A) $1,500 B) $2,000 C) $0 D) $1,000

D SMA is a line of credit with one restriction: it may not be used if account equity would fall below minimum maintenance. In this account, maintenance equity is $3,000 (25% of $12,000),0 and the current equity in the account is $4,000 ($12,000 MV − $8,000 DB). Therefore, only $1,000 may be withdrawn to keep the current equity at the minimum of $3,000.

Stabilizing bids may be entered at A) whatever stabilizing price is stated in the prospectus. B) any reasonable price necessary to support the public offering price. C) a price not exceeding 5% above the public offering price. D) a price no higher than the public offering price.

D Stabilizing bids cannot be used to raise the market price of an issue. Stabilization may only be used to support a new issue security at or below the public offering price.

A corporation must have stockholder approval to A) declare a cash dividend. B) declare a 15% stock dividend. C) repurchase 100,000 shares of stock for its Treasury. D) issue convertible bonds

D Stockholders are entitled to vote on the issuance of additional securities that would dilute shareholders' equity (the shareholders' proportionate interest). Conversion of the bonds would cause more shares to be outstanding, thus reducing the proportionate interest of current stockholders. Decisions that are made by the board of directors and do not require a stockholder vote include the repurchase of stock for its Treasury, declaration of a stock dividend, and declaration of a cash dividend.

Trading in expiring options series concludes the same day as expiration at A) 11:59 pm ET. B) 10:59 pm CT. C) 5:30 pm ET. D) 4:00 pm ET.

D The last time an option can be traded (bought or sold) is 4:00 pm ET on the third Friday of the expiration month (the expiration date). Don't confuse that with the latest time an exercise notice may be submitted. That time is 5:30 pm ET on the expiration date. Think conceptually: you must own an option before you can exercise it. If you buy an option at the last possible time (4:00 pm), you have 1 1/2 hours (up to 5:30 pm) to submit the notice to exercise it. Finally, the official expiration time is 11:59 pm ET or 10:59 pm CT on the expiration date.

A customer buys 100 shares of ABC at 56.50 and writes 1 ABC Aug 60 call at 2. If the call is exercised, the consequences are a cost basis of $56.50 per share. a cost basis of $58.50 per share. sales proceeds of $60 per share. sales proceeds of $62 per share. A) I and III B) II and III C) II and IV D) I and IV

D The premium of the option affects the basis of the stock (bought or sold) as a result of exercise, adding the premium per share ($2) to the price per share ($60), for total sales proceeds of $62. The original cost basis is not affected by the exercise, so it remains $56.50.

A qualified legal opinion issued for a municipal bond underwriting means that A) the revenue bond issue has certain debt limitations. B) the bond attorney is qualified to express her opinion on the bond. C) the bond counsel is considered competent. D) the legal opinion is qualified with restrictions and conditions.

D The word qualified describes the legal opinion, not the attorney (or bond counsel) who issued it. A qualified legal opinion is one in which the bond counsel expresses reservations about conditions that may affect the bond's status. An unqualified legal opinion is rendered without restriction or condition.

The working capital of a corporation includes all of the following except A) cash. B) accounts receivable. C) marketable securities of other companies. D) convertible bonds.

D The working capital of a corporation is equal to its current assets minus its current liabilities. A current liability is payable within 12 months. Because convertible bonds are long-term (not short-term) liabilities, they are not included as working capital.

Your firm has just assigned you a new client. Wanting to do the best job you can, you review the current investment holdings of the client. Included are the following mutual fund accounts: $50,000 in Class B shares of the STU Growth Fund $25,000 in Class A shares of the STU International fund $15,000 in Class A shares of the TUV Utility Fund STU and TUV offer rights of accumulation and breakpoints at $25,000, $50,000, and $100,000. If the client wishes to invest $20,000 into the Class A shares of the TUV Technology Fund, the sales charge would be based on A) the $100,000 breakpoint. B) the next computed net asset value. C) the $50,000 breakpoint. D) the $25,000 breakpoint.

D There are several important points in this question. Rights of accumulation provide that a new purchase is added to the value of existing accounts to determine the breakpoint reached. However, only Class A shares count because the Class B shares never paid a front-end load. Of course, only shares in the same fund family are used - we cannot combine STU shares with TUV shares. As far as the math, we have $15,000 in one TUV fund and are adding another $20,000. That brings the investor's account in TUV funds to $35,000, which is enough to meet the $25,000 breakpoint. Finally, the sales charge is computed as a percentage of the public offering price (POP), not the NAV.

A customer bought a 10% interest in a real estate limited partnership by investing $100,000. The partnership buys a $4 million property with the funds, making a down payment of $800,000 and financing the balance with a nonrecourse mortgage of $3.2 million. Subsequently, the partnership cannot meet the mortgage payment; the lender forecloses when the remaining mortgage balance is $3 million, auctioning off the property for $1 million. How much of the investment will the customer recover? A) $32,000 B) $100,000 C) $10,000 D) $0

D There is a lot more information in this question than necessary. Simply put, the deal went bankrupt—the asset was sold for less than the mortgage. That means the investor's $100,000 is totally lost.

An options investor wishing to follow a market-neutral strategy would be most likely to find which of the following most appropriate? A) A long broad index call B) A long straddle C) A debit put spread D) A time spread

D Time spreads, also called calendar or horizontal spreads, consist of two options of the same type with the same strike price, but different expiration months. The strategy expects the market to stay relatively level. The profit arises from the time decay of the later expiration date. A long straddle is profitable only if there is market movement. The same is true with the long call - the market price must go up. A debit put spread is a bearish strategy, so this strategy requires the market price to decline.

A confirmation to a customer purchasing a new issue of bonds must disclose all of the following except A) customer's name. B) coupon rate and maturity date. C) settlement date. D) current yield.

D To conform with industry rules, confirmations must include the customer's name, trade and settlement dates, coupon rate and maturity, and the yield to maturity or yield to call (whichever is lower). The current yield (annual interest / current market price) is not included on confirmations.

It is generally understood that the least complicated employer-sponsored retirement plan is the Savings Incentive Match Plan for Employees (SIMPLE). These plans tend to have certain restrictions. Among them are the restriction that A) employer matching contributions are made with after-tax funds. B) the catch-up provision for those 50 and older is limited to $1,000. C) there must be fewer than 100 employees who earned at least $5,000 during the preceding calendar year. D) the business cannot have another retirement plan in place.

D To institute a SIMPLE plan, the business cannot have any other retirement plan in place. The limit is 100 or fewer, not fewer than 100. The catch-up provision is $3,000, and both employee and employer contributions are made with pre-tax funds.

One of your customers with a JTWROS account contacts you to remove the other tenant and put the account into the customer's own name. This can be done only A) if you contact the other tenant and get their approval. B) if the customer has a full power of attorney over the account. C) upon the death of the other tenant. D) if the change has been authorized by a qualified and registered principal designated by the member.

D Under FINRA rules, no change in any account name(s) can be made unless the change has been authorized by a qualified and registered principal designated by the member. This principal must, before giving her approval of the account designation change, be personally informed of the essential facts relative thereto and indicate her approval of such change in writing. The essential facts relied upon by the person approving the change must be documented in writing and preserved with the customer account records. One of those facts is approval of the other tenant, but that approval goes to the principal, not to you, the registered representative. Even in the case of death of the other tenant, the principal needs to see the proper documentation, such as a death certificate.

Which of the following assets would be least likely used to back a collateralized debt obligation (CDO)? A) Auto loans B) Corporate receivables C) Credit card debt D) Mortgages

D Unlike CMOs, which are backed by mortgages (as the M indicates), CDOs are invariably backed by some other form of asset. Remember that what someone owes is their debt, while it is an asset to the creditor.

Which of the following choices best describes the formula to determine earnings per share? A) Retained earnings divided by the number of shares of common issued and outstanding B) (Net income + preferred dividends) divided by (the number of shares of common issued − the number of shares of treasury stock) C) (Net income − preferred dividends) divided by the number of shares of common authorized D) [EBIT − (interest + taxes)] − preferred dividends] divided by the number of shares of common outstanding

D We calculate earnings per share (EPS) by subtracting any preferred dividends from the net income and dividing that by the number of shares of common stock outstanding. EBIT represents earnings before interest and taxes. By subtracting interest expenses and taxes, the company's net income is determined. It is the outstanding shares that are used, not the authorized. Although the issued minus the treasury stock equals the outstanding shares, the preferred dividend is subtracted from the net income, not added to it.

The term trading flat means A) the bond is in default. B) the bond is sold without markup or commission. C) the price of the bond has remained level. D) there is no accrued interest.

D When a bond trades flat, the buyer does not owe accrued interest to the seller. Trading flat means there is no accrued interest due. While it is true that a bond in default trades flat, one cannot say that the term trading flat means the bond is necessarily in default.

Due to a distribution of stock, the contract size in the JGH Oct 50 call options is 108. A customer purchasing one of these contracts for a premium of 2½ would expect to pay A) $250. B) $330. C) $258. D) $270.

D With a contract size of 108 shares (likely from an 8% stock dividend) and a premium of $2.50 per share, the total cost is $270. Regardless of the reason for the contract size being other than 100 shares, the price paid for an option is always the premium multiplied by the number of shares in the contract. In this question, that would be a premium of $2.50 per share (2½) times 108 = $270.00.

Which of the following statements regarding the Federal Farm Credit System securities are not true? A) They issue short-term notes and long-term bonds. B) The proceeds are used to make loans to farmers. C) Interest is tax exempt at the state and local levels. D) They are direct obligations of the U.S. government.

D With the exception of Ginnie Mae, all agency securities are indirect obligations of the U.S. government.

An investor looking for current income while wishing to reduce interest rate risk would most likely find which of the following investments suitable? A) A U.S. Treasury note maturing in eight years B) A $100 par preferred stock callable at 102 in three years C) A bond unit investment trust (UIT) with a duration of five years D) U.S. Treasury STRIPS maturing in five years

One of the features of a unit investment trust is that it has a defined end date. The bonds held in the UIT in our question all mature in five years. Regardless of how high current market interest rates rise, bonds pay off at face value when they mature. The longer the investor has to wait for maturity, the greater the interest rate risk. That makes the Treasury notes a less acceptable choice. You can assume that a callable security, preferred or debt, is not going to be called unless interest rates go down. Remember, when interest rates decline, fixed income investment rise in price so the investor would not want the stock called away. The STRIPs will mature in five years but, as zero coupon securities, will pay no income in the interim.


Set pelajaran terkait

11.1 - 11.7 Security Assessments

View Set

Theory and Practice of Counseling and Psychotherapy - Midterm

View Set

Chapter 24 Fluid, Electrolyte, and Acid-Base Balance

View Set

Blood Vessels and Circulation Anatomy II Exam 1

View Set